BOC Exam 1, 3, 4, 5

Ace your homework & exams now with Quizwiz!

An athlete has an avulsion fracture of the distal lateral malleolus. What is the most likely mechanism of injury? Choose one: a. Supination b. Pronation c. Plantarflexion d. Inversion e. Eversion

d. Inversion

Which of the following is the most common mechanism of injury for a brachial Plexus neurapraxia? Choose one: a. Cervical hyperextension with cervical compression b. Cervical hyperextension with a rotational force c. Forward cervical flexion with an axial load d. Lateral cervical flexion with contralateral shoulder depression e. Lateral cervical flexion with contralateral shoulder elevation

d. Lateral cervical flexion with contralateral shoulder depression

An organization sponsoring a state wide athletic tournament does not have a written emergency action plan. Which of the following legal consequences would impact the organization/ Choose one: a. Malfeasance b. Misfeasance c. Neglect d. Negligence e. Tort

d. Negligence

Which disorder is based on persistent thoughts, impulses, or images about exaggerated or imaginary circumstances and results in repetitive behaviors? Choose One: a) Attention Deficit Hyperactivity Disorder b) Eating disorder c) Mood disorder d) Obsessive Compulsive Disorder e) Post-traumatic Stress Disorder

d. Obsessive Compulsive Disorder

Which of the following terms refers to the failure of an athletic trainer to do something that should have been done under normal circumstances? Choose one: a. Breach of contract b. Commission c. Malpractice d. Omission

d. Omission

Which of the following modalities transfers thermal energy through the process of conduction? Choose all that apply: a. Warm whirlpool b. Ultrasound c. Shortwave diathermy d. Paraffin bath e. Moist heat pack

d. Paraffin bath e. Moist heat pack

Which of the following is the optimal positioning of the glenohumeral joint to palpate the greater tuberosity of the humerus? Choose One: a. Passive abduction b. Passive adduction c. Passive flexion d. Passive internal rotation e. Passive external rotation

d. Passive internal rotation

During a full contact practice, a football player remains supine following a collision. Which of the following steps should the athletic trainer take first when completing an on-field evaluation? Choose one: a. Assess the athlete's level of circulation b. Establish or maintain an open airway for the athlete c. Look. Listen, and feel for any sign of the athlete's breathing d. Stabilize the athlete's cervical spine and determine the athlete's level of consciousness

d. Stabilize the athlete's cervical spine and determine the athlete's level of consciousness

An athletic trainer is engaged in an irresolvable dispute with a basketball coach regarding injury management of an athlete. Which of the following courses of action is most appropriate? Choose one: a. Discuss the situation with colleagues at the institution and formulate an action plan to resolve the dispute b. Discuss the situation with peers at other institutions and formulate an action plan to resolve the dispute c. Assign another athletic trainer to work with the basketball team d. Utilize the "chain of command" as defined by the organizational hierarchy of the institution e. Write a memo to the national coaching association outlining the coach's behavior

d. Utilize the "chain of command" as defined by the organizational hierarchy of the institution

How often should personnel involved in implementing an emergency action plan be educated on their roles? Choose One: a) Weekly b) Monthly c) Quarterly d) Bi-annually e) Annually

e) Annually

Which of the following symptoms must resolve prior to an athlete's return to play after an upper respiratory infection? Choose One: a) Nasal congestion b) Sore throat c) Fatigue d) Cough e) Fever

e) Fever

Which of the following is an epiphyseal condition that causes tissue necrosis and potential deformation to the head of the femur? Choose One: a) Sever's disease b) Scheuermann's disease c) Os trigonum syndrome d) Osgood-Schlatter disease e) Legg-Calve-Perthes syndrome

e) Legg-Calve-Perthes syndrome

An athlete is two weeks post-surgery for an ACL reconstruction. The athlete has controlled inflammation and has been cleared to move to the next phase of rehabilitation. Which of the following activities would be most appropriate at this time? Choose One: a) Lateral Lunges b) Lateral Step-Ups c) Leg Extensions d) Lunges e) Mini Squats

e) Mini Squats

During a pre-participation examination, a 14-year-old athlete presents with a blood pressure of 146/95 mmHg. Which of the following actions should an athletic trainer take? Choose One: a) Allow only conditioning and strength training activities at this time b) Allow only conditioning activities at this time c) Contact the athlete's coach d) Counsel the athlete on approved methods to reduce hypertension in athletes e) Refer the athlete to the appropriate healthcare provider for a follow-up evaluation

e) Refer the athlete to the appropriate healthcare provider for a follow-up evaluation

An athletic trainer is providing services at a downhill skiing competition. A skier falls and hits her head after clearing a jump, and then she skis to the athletic trainer complaining of pain along her cervical spine with numbness in her right upper extremity. In addition, the skier reports nausea and a moderate headache. What action should the athletic trainer perform first? Choose One: a) Assess cranial nerves b) Evaluate the athlete's cervical range of motion c) Evaluate the athlete for a concussion d) Refer the athlete to a physician e) Stabilize the athlete's cervical spine

e) Stabilize the athlete's cervical spine

Which of the following medications is most appropriate in the treatment of tinea pedis? Choose One: a) Oral antipyretic b) Oral antibiotic c) Topical antibiotic cream d) Topical antimicrobial cream e) Topical antifungal cream

e) Topical antifungal cream

An athletic trainer modifies a football player's pads so they better fit the athlete. During practice, the athlete is injured and subsequently the athlete's family blames the athletic trainer and the institution for inadequate/improper care of the athlete. What legal principle is illustrated? Choose One: a) Accident b) Assumption of risk c) Product liability d) Sovereign immunity e) Tort

e) Tort

With the patient seated, which of the following is the optimal positioning of the glenohumeral joint to palpate the greater tuberosity of the humerus? Choose One: a) passive abduction b) passive adduction c) passive flexion d) passive internal rotation e) passive external rotation

e) passive external rotation

An athletic trainer is treating a tennis player for lateral epicondylitis and decides to administer therapeutic ultrasound. Which of the following parameters would be most appropriate in decreasing pain and increasing blood flow to the treatment area during a five minute treatment? Choose one: a. 0.5w/cm"2; 1MHz b. 1.0w/cm"2; 1MHz c. 1.5 w/cm"2; 1MHz d. 0.5w/cm"2; 3MHz e. 1.0 w/cm"2; 3MHz

e. 1.0 w/cm"2; 3MHz

Which type of fracture occurs due to the pulling force of an attached tendon causing a fragmented bone? Choose one: a. Spiral b. Oblique c. Greenstick d. Comminuted e. Avulsion

e. Avulsion

A positive Romberg test most likely indicates a deficiency in which area? Choose one: a. Vision b. Strength c. Motor function d. Hearing e. Balance

e. Balance

A male athlete reports to the athletic trainer expressing concern for recent painful urinations with traces of blood and purulent discharge. There has been no recent trauma. To whom should the athletic trainer refer this athlete to for further examination? Choose one: a. Oncologist b. Proctologist c. Hematologist d. Gastroenterologist e. Family practice physician

e. Family practice physician

Which movement is increased when performing a hip posterior glide (anterior to posterior)? Choose all that apply: a. Abduction b. Adduction c. Extension d. External rotation e. Flexion f. Internal rotation

e. Flexion f. Internal rotation

Which vitamin is involved in activation of blood clotting factors? Choose one: a. A b. C c. D d. E e. K

e. K

Which condition often associated with excessive pronation is most likely a predisposing factor for the development of overuse injuries? Choose one: a. Hallux valgus b. Hammer toes c. Metatarsalgia d. Pes Cavus e. Pes Planus

e. Pes Planus

When attempting to palpate the dome of the talus, the athletic trainer should place the athlete's foot in which position? Choose none: a. Dorsiflexion b. Eversion c. Inversion d. Neutral e. Plantarflexion

e. Plantarflexion

During the assessment of a 16 year old athlete, an athletic trainer notes the following information • Pulse: 76 bpm and regular • Blood pressure: 116/72 mmhg • Pupils: equal and reactive to light • Respiration: 26 breaths per min • Body temp: 97.9 degrees F (36.6 degrees C) Which measurement is considered abnormal for this athlete? Choose One: a. Blood pressure b. Body temperature c. Pulse d. Pupils e. Respiration

e. Respiration

A patient presents with pain and swelling in the anatomical snuff box. What condition should the athletic trainer suspect? Choose one: a. Boxer's fracture b. Capitate dislocation c. De Quervain's disease d. Lunate dislocation e. Scaphoid fracture

e. Scaphoid fracture

Which type of fracture occurs from a rotational force? Choose one: a. Avulsion b. Comminuted c. Greenstick d. Oblique e. Spiral

e. Spiral

Which of the following treatments is most appropriate for Morton's Neuroma? Choose One: a) Commercially made gel heel cup to support the posterior calcaneus b) Full length gel arch support c) Adhesive foam pad secured by an arch taping in the medial longitudinal arch d) Felt donut pad over the medial aspect of 1st metatarsal e) Teardrop shaped foam pad on the plantar surface between the 3rd and 4th metatarsals

e. Teardrop shaped foam pad on the plantar surface between the 3rd and 4th metatarsals

Which myofascial technique is most appropriate to treat scar tissue adhesions? Choose one: a. Diagonal myofascial release b. Ischemic compression c. Petrissage d. Skin rolling e. Transverse friction massage

e. Transverse friction massage

An athlete who has been suffering from an addictive behavior is now ready to seek assis-tance. Which of the following definitions match to the correct stage of readiness? A. Follow-up B. Action C. Contemplation D. Preparation E. Pre-Contemplation 1. Behavioral changes have begun 2. Strong thought is given to change a behavior 3. Relapse is a primary concern 4. Consideration to change has not been done 5. A decision to to change behavior is made

• Action: Behavioral changes have begun • Contemplation: Strong thought is given to change a behavior • Follow-Up: Relapse is a primary concern • Pre-Contemplation: Consideration to change has not been done • Preparation: A decision to to change behavior is made

Following an athlete's concussion, the team physician has given the athletic trainer instruction to proceed through graduated concussion rehabilitation return to play progression. What are the appropriate ordinal steps to follow? A. Step1 B. Step 2 C. Step 3 D. Step 4 E. Step 5 F. Step 6 1. Active Recovery 2. Full-Contact Practice 3. Light aerobic exercise 4. Noncontact training drills 5. RTP 6. Sport Specific Exercise

• Active Recovery: Step 1 • Light aerobic exercise: Step 2 • Sport Specific Exercise: Step 3 • Non-contact training drills: Step 4 • Full-Contact Practice: Step 5 • RTP: Step 6

Which of the following items are classified in these purchasing categories? A. Annual re-purchase B. Capital equipment C. Consumable supplies 1. Adhesive bandage 2. Ambulance contract 3. Athletic tape 4. Electrical stimulation unit 5. Ice machine 6. injury tracking software

• Adhesive bandage: consumable supplies • Ambulance contract: annual repurchase • athletic tape: consumable supplies • Electrical stimulation unit: capital equipment • Ice machine: capital equipment • injury tracking software: annual repurchase

When writing a SOAP note, abbreviations are common. Which of following abbreviations are utilized below? A. BID B. QD C. P D. P.O. E. PRN F. RX 1. After 2. As needed 3. Once daily 4. Prescription 5. Take medication orally 6. Twice daily

• After: P • As needed: PRN • Once daily: QD • Prescription: RX • Take medication orally: P.O. • Twice daily: BID

What are the signs and symptoms of the following exertional heat related conditions? A. Heat cramps B. Heat Exhaustion C. Heat stroke 1. Altered neuromuscular control 2. Decreased sweating response 3. Disorientation or confusion 4. Markedly increased body temperature 5. Muscle spasms 6. Normal or slightly lowered body temperature 7. Profuse sweating 8. Rapid weak pulse

• Altered neuromuscular control: Heat cramps • Decreased sweating response: heat stroke • Disorientation or confusion: heat stroke • Markedly increased body temperature: heat stroke • Muscle spasms: heat cramps • Normal or slightly lowered body temperature: heat exhaustion • Profuse sweating: heat exhaustion • Rapid weak pulse: heat exhaustion

74. In reference to the talus bone in anatomical position, where are the locations of the following structures? A. Calcaneonavicular ligament B. Calcaneus C. Dorsal pedal artery D. Personeal brevis tendon E. Tibia 1. Anterior 2. Inferior 3. Medial 4. Lateral 5. Proximal

• Anterior: Dorsal pedal artery • Inferior: calcaneus • Medial: Calcaneonavicular ligament • Lateral: Peroneal brevis tendon • Proximal: Tibia

Various massage types are used in conjunction with therapeutic exercise to assist with soft tissue mobilization. What are the characteristics of of the following massage techniques? 1. Combination of compression with broadening and muscle stripping 2. Deep pressure cross fiber stroking 3. ischemic compression 4. lifting and kneading 5. Rapid shaking 6. Stroking of the skin 7. Tapping or pounding

• Combination of compression with broadening and muscle stripping: active assisted • Deep pressure cross fiber stroking:Friction • ischemic compression: neuromuscular • lifting and kneading: petrissage • Rapid shaking: vibration • Stroking of the skin: effleurage • Tapping or pounding: tapotement

The athletic trainer is assigned to complete a dermatological assessment at a store high school wrestling tournament. Which of the following characteristics describe the skin pathology? A. Clear fluid-filled vesicle B. Elevated reddened pustule C. Honey colored vesicle D. Scaling ring-shaped plaque E. Smooth dome-shaped papule 1. Contact dematitis 2. Furnuncle 3. Impetigo contagiosa 4. Molloscum contagiosum 5. Tinea corporis

• Contact dematitis: clear fluid filled vesicle • Furnuncle: elevated reddened pustule • Impetigo contagiosa: honey colored vesicle • Molloscum contagiosum: smooth dome shaped papule • Tinea corporis: scaling ring shaped plaque

Which abdominal quadrant is each internal organ located? A. Appendix B. Bladder C. Colon D. Heart E. Liver F. Lung G. Pancreas H. Spleen 1. Lower Left Q 2. Lower Left Q 3. Lower Right Q 4. Lower Right Q 5. Lower Right Q 6. Upper Left Q 7. Upper Left Q 8. Upper Left Q 9. Upper Right Q 10. Upper Right Q

• Lower Left Q: Bladder • Lower Left Q: Colon • Lower Right Q: Colon • Lower Right Q: Bladder • Lower Right Q: Appendix • Upper Left Q: Spleen • Upper Left Q: Lung • Upper Left Q: Heart • Upper Right Q: Liver • Upper Right Q: Pancreas

Which of the following descriptions matches the budget method? Match the options from the toolbar on the top to the targets listed in the table on the bottom. Each option can be used once. Budget Method: Zero- Based Variable Performance Lump-Sum Line- Item Fixed Description - Provides budgets for separate categories of expenditures - Provides fixed amount of money and freedom to spend it - Requires expenditures be adjusted according to revenues - Requires justification for each expense without reference to previous spending patterns - Requires list of expenditures for specific categories that cannot be transferred for other use - Requires projection of expenditures and program - income on a monthly basis to determine yearly budget

- PROVIDES BUDGETS FOR SEPARATE CATEGORIES OF EXPENDITURES: PERFORMANCE - PROVIDES FIXED AMOUNT OF MONEY AND FREEDOM TO SPEND IT: LUMP-SUM - REQUIRES EXPENDITURES BE ADJUSTED ACCORDING TO REVENUES: VARIABLE - REQUIRES JUSTIFICATION FOR EACH EXPENSE WITHOUT REFERENCE TO PREVIOUS SPENDING PATTERNS: ZERO-BASED - REQUIRES LIST OF EXPENDITURES FOR SPECIFIC CATEGORIES THAT CANNOT BE TRANSFERRED FOR OTHER USE: LINE-ITEM - REQUIRES PROJECTION OF EXPENDITURES AND PROGRAM INCOME ON A MONTHLY BASIS TO DETERMINE YEARLY BUDGET: FIXED

An injury to C5 would likely elicit weakness in which shoulder movement? A. Abduction B. Adduction C. External rotation D. Internal rotation E. Extension

A. Abduction

What are the common causes of iron deficiency in athletes? Choose all that apply A. Abnormal menstruation B. Adequate fluid intake C. Consuming fruits and vegetables D. Consuming meats E. Consuming vitamin and mineral supplements F. Gastrointestinal bleeding G. Hematuria H. Profuse sweating from long-duration training sessions

A. Abnormal menstruation F. Gastrointestinal bleeding G. Hematuria H. Profuse sweating from long-duration training sessions.

An athlete with which of the following conditions would NOT be allowed to participate in football? Choose all that apply A. Absence of one kidney B. Alopecia C. Asthma D. Atlantoaxial instability E. Inguinal hernia F. Sickle cell trait G. Undescended testicle

A. Absence of one kidney D. Atlantoaxial instability

During football practice, linebacker suddenly collapses and is unconscious. As the responding athletic trainer, what initial action should be taken? A. Activate EMS B. Administer oxygen C. Apply AED D. Begin CPR E. Place athlete on spine board

A. Activate EMS

What are the contractions of joint mobilization therapy? Choose all that apply A. Acute inflammation B. Breaking up adhesions and relieving capsular restrictions C. Distracting impacted tissues D. Hypermobility E. Infection F. Reducing pain and muscle tension G. Rheumatoid arthritis H. Restoring full ROM and facilitating healing

A. Acute inflammation D. Hypermobility E. Infection G. Rheumatoid Arthritis

Which of the following are contraindications for heat modalities? Choose all that apply A. Acute injuries B. Chronic muscle spasm C. Closed infections D. Deep vein thrombosis E. Joint contractures F. Joint effusion

A. Acute injuries C. Closed infections D. Deep vein thrombosis

What is the PRIMARY purpose of organzation's policy and procedures manual? A. Address the common needs of the organ-ization and its employees B. Educate the employees on healthcare benefits and retirement options C. Outline effective communication strate-gies for the organization and employees D. Outline the fiscal management philoso-phies of the organization

A. Address the common needs of the organ-ization and its employees

Which of the following medical condition are combines in the female athlete triad? Choose all that apply A. Amenorrhea B. Depression C. Disordered eating D. Dysmenorrhea E. Mittelschmerz F. Olgiomenorrhea G. Osteoporosis

A. Amenorrhea C. Disordered eating G. Osteoporosis

What is the appropriate home care advice for a high school athlete who suffered a mild ankle sprain during practice today? Choose all that apply A. Ankle elevation above heart B. Crutch use if painful to bear weight on injured ankle C. Cryotherapy D. Elastic bandage from ankle joint to mid calf E. Elastic bandage from tip of toes to mid calf F. Hot tub at home G. Thermotherapy H. Wheelchair

A. Ankle elevation above heart B. Crutch use if painful to bear weight on injured ankle C. Cryotherapy E. Elastic bandage from tip of toes to mid calf

Which of the following cardiac conditions is MOST often associated with Marfan syndrome? A. Aorta abnormalities B. Hypertrophic cardiomyopathy C. Mitral valve prolapse D. Myocarditis E. Tricuspid valve abnormalities

A. Aorta abnormalities

Which of the following signs and symptoms are found in a person suffering from anxiety disorder? Choose all that apply A. Appetite disturbance B. Failure to follow directions C. Fear of losing control D. Intense fear of being overweight E. Irritability F. Giving away personal items G. Recurrent and persistent thoughts H. Sleep impairment

A. Appetite Disturbance C. Fear of losing control E. Irritability H. Sleep Impairment

Which of the following are legal interview questions that could be asked during a job interview? Choose all that apply A. Are you able to perform the essential job functions of this position? B. Are you available to work on the weekends? C. Are you married? D. Do you have any illnesses or disabilities? E. Do you have verifications of your legal right to work in the United States? F. Do you plan to start a family this year?

A. Are you able to perform the essential job functions of this position? B. Are you available to work on the weekends? E. Do you have verifications of your legal right to work in the United States?

Which legal defense protects an ATC and places responsibility on the athlete for understanding the risks associated with participation in a sport? A. Assumption of risk B. Comparative negligence C. Good Samaritan immunity D. Sovereign immunity E. Statute of limitations

A. Assumption of Risk

Athlete who is seeking second opinion wants medical records sent to new doctor. What procedure is appropriate to ensure privacy? A. Athlete must sign an authorization for release of medical information B. Athlete must sign an informed consent statement C. Athlete must present insurance information D. Athlete must fill out a new patient information sheet E. Athlete must complete an insurance referral request form

A. Athlete must sign an authorization for release of medical information

What mechanisms of injury are associated with cervical spine fractures in football? Choose all that apply A. Axial load B. Hyperextension C. Hyperflexion D. Lateral flexion E. Rotation

A. Axial load C. Hyperflexion

Which of the following will help prevent hypoglycemia during exercise? Choose one : A. Carbohydrate drinks B. Coffee C. Protein shakes D. Vitamin water E. Water

A. Carbohydrate drinks

During PPE, a wrestler presents with small red colored smooth domed papules on food. What medical specialist would be MOST appropriate to refer to? A. Dermatologist B. ER physician C. General medical physician D. Podiatrist

A. Dermatologist

Which of the following events occurs in the proliferation stage of wound healing? A. Development of fibrous tissue B. Leukocyte migration C. Platelet accumulation D. Resolution of matrix

A. Development of fibrous tissue

Which of the following describes the signs and symptoms of heat stroke? Choose all that apply A. Disorientation B. Red/dry skin C. Rapid strong pulse D. Rapid weak pulse E. Cool clammy skin F. Body temperature 102 F

A. Disorientation B. Red/dry skin C. Rapid weak pulse

An athletic trainer is contracted to work the district cross country high school championships. Which of the following signs would identify an acute asthma attack? Choose all that apply A. Dry cough B. Excessive perspiration C. Flushed skin D. Irritability E. Labored breathing F. Wheezing

A. Dry cough E. Labored breathing F. Wheezing

Which of the following criteria describes the accurate fit of a football helmet? A. Ear holes line up with the ear lobes B. The cheek pads fit snugly onto the cheeks C. The external occipital protuberance is palpable just below the base of the helmet D. The front of helmet covers the eyebrows E. There is visible recall with downward pressure on top of helmet

A. Ear holes line up with the ear lobes

Which of the following should be considered when preparing an emergency action plan for an athletic facility? Choose all that apply A. Emergency equipment required for site activities B. The EAP should be practiced weekly at a min-imum C. The facility address, directions to facility and land line phone numbers D. The personnel, job descriptions and salary E. The personnel, their qualifications, and role in emergent situation F. The site mascot, colors, and previous records held in athletics

A. Emergency equipment required for site activities C. The facility address, directions to facility and land line phone numbers E. The personnel, their qualifications, and role in emergent situation

Scenario: 28 y/o female ice hockey ath has horizontal laceration to left mid-anterior forearm. 2 teammates control bleeding and are covered in significant amount of blood. Eval of ath reveals 7cm wide 2cm deep laceration, BP 98/54 mmHg, pulse 115 bpm, rapid and shallow 24 breaths per minute, pale skin, conscious by lethargic. Question: Ath is transported to ED where they find ath is positive for HBV. Which steps should athletic trainer take upon learning of the athlete's acute HBV diagnosis? A. Ensure the exposed team members have up to date HBV vaccines B. Inform team members that the athlete is positive for HBV C. Inform the state health department of the HBV outbreak D. Require the all team members are tested for HBV prior to next competition

A. Ensure the exposed team members have up to date HBV vaccines

What type of knee brace would a physician prescribe following return to sport after successful ACL reconstruction surgery? A. Functional knee brace B. Patellofemoral knee brace C. Prophylactic knee brace D. Rehabilitative knee brace

A. Functional knee brace

When performing electrical muscular stimulation on a post surgical ACL reconstruction athlete, which of the following would produce tetany of the quadriceps? A. High frequency 40-50 pps B. Low frequency 5-15 pps C. Medium frequency 15-30 pps D. Very high frequency 150-200 pps

A. High frequency 40-50 pps

Which of the following positions is the most appropriate for the athlete when the athletic trainer is palpating the spleen? A. Hook lying position B. Left side lying C. Right side lying D. Supine with legs and arms extended

A. Hook lying position

Which of the following modalities could be utilized to reduce pain in an acute muscle spasm? Choose all that apply A. Ice pack B. Microcurrent C. Paraffin bath D. Russian E. TENS

A. Ice pack E. TENS

Which of the following budgeting classifications requires justification only for expenditures that exceed the previous budget period? A. incremental budget B. line item budget C. planning programming budget D. spending reduction model E. zero-based budget

A. Incremental budget

Visual acuity of 20/50 is indicative of which of the following? A. Individual can read at 20 feet what aver-age person can read at 50 feet B. Individual can read the 20 items listed 50 feet away C. Individual can read the 50 items listed 20 feet away D. Individual can read 20 percent of items at 50 feet away E. Individual can read at 50 feet what an av-erage person can read at 20 feet away

A. Individual can read at 20 feet what aver-age person can read at 50 feet

Based on Gate Control Theory, what TENS parameters for MOST appropriate for pain relief? A. Intensity: sensory, 80 pps, 75 microsec-onds B. Intensity: motor, 80 pps, 75 microsec-onds C. Intensity: sensory, 15 pps, 150 microsec-onds D. Intensity: motor, 4 pps, 200 microsec-onds E. Intensity: sensory, 20 pps, 50 microsec-onds

A. Intensity: sensory, 80 pps, 75 microsec-onds

Which of the following rotator cuff therapeutic strengthening exercises would be appropriate two weeks post operation for an arthroscopic slap lesion repair? A. Isometric B. Isotonic C. Isokinetic D. Open kinetic chain E. Plyometric

A. Isometric

During PPE, 22 year old athlete reports mild fatigue, systemic body aches, dyspnea, pitting edema, palpitations. What condition should the ATC suspect? A. Myocarditis B. Mononucleosis C. Sickle cell anemia D. Pulmonary embolus E. Deep vein thrombosis

A. Myocarditis

The suffix "aglia" is indicative of what condition? A. Pain B. Softening C. Swelling D. Tumor E. Vomitting

A. Pain

Which of the following is true regarding preferred provider organizations? A. Provide discounted healthcare and have specified provider networks B. Provide emergency measures only and limit the location of treatment C. Provide preventative healthcare only and limit location of treatment D. Restrict number and types of providers that give treatment

A. Provide discounted healthcare and have specified provider networks

Inferior surface of the talus articulars with which of the following bones? A. calcaneous B. cuboid C. metatarsal D. navicular E. tibia

A. calcaneous

71. After assessing an athlete with a suspected femoral fracture, the athlete appears pale, is perspiring, has a rapid and weak pulse, and has rapid and shallow respirations. Which physiological response is most likely occurring? Choose One: a) 1.0 wcm"2 1MHz for 5 minutes continuous b) 1.0 wcm"2 3 MHz for 5 minutes continuous c) 1.5 wcm"2 1MHz for 12 minutes continuous d) 1.5 wcm"2 3 MHz for 8 minutes continuous e) 2.0 wcm"2 1MHz for 3 minutes continuous

Answers and question don't line up

A 19 year old basketball player was poked in the eye with a finger during a rebound. The player complains of pain, diplopia, floaters, and photophobia. Upon inspection, the athletic trainer notes the following: Subconjunctival redness • Pupils equid and reactive • Excessive tearing • Eye tracking normal The athlete has recovered from his injury and his vision is checked using a Snellen eye chart. The athletes vision is 20/50. What does this assessment mean? Choose one: a. Athlete is able to see at 50 ft (1524 cm) what most patients can see at 20 ft (610 cm) b. Athlete is able to see at 20 ft (610 cm) what most patients can see at 50 ft (1524 cm) c. Athlete can read 50 items at 20 ft (610 cm) d. Athlete can read 20 items at 50 ft (1524 cm)

Athlete is able to see at 20 ft (610 cm) what most patients can see at 50 ft (1524 cm)

What percentage of a patients body weight is made up of water? A. 70-90% B. 50-70% C. 30-50% D. 10-30% E. 0-10%

B. 50-70%

An athlete sprained his ankle during a soccer match and needs to be fitted for crutches. Which of the following are correct strategies to fit this athlete properly? Choose all that apply A. Adjust crutch height to 4 finger widths below axilla B. Adjust crutch height to 2 finger widths below axilla C. Adjust hand grip so elbow is flexed 10-15 degrees D. Adjust hand grip so elbow is flexed 25-35 degrees E. Place crutch tip even with the longitudinal arch F. Place crutch tip two in in front six in to the side of the sole of the shoe

B. Adjust crutch height to 2 finger widths below axilla D. Adjust hand grip so elbow is flexed 25-35 degrees F. Place crutch tip two in in front six in to the side of the sole of the shoe

A teammate informs the athletic trainer that an athlete is Type 1 diabetic and did not eat breakfast before the workout. The athletic trainer determines the athlete is hypoglycemic. What IMMEDIATE course of action should the athletic trainer take? Choose all that apply A. Activate EMS B. Administer initial 0.35-0.53 ox (10-15 g) of fast-acting carbohydrate C. Administer fast-acting insulin injection D. Measure blood glucose level E. Prepare glucagon kit and administer glucagon injection

B. Administer initial 0.35-0.53 ox (10-15 g) of fast-acting carbohydrate D. Measure blood glucose level

Which of the following conditions comprise the female athlete triad? A. ACL injury, olgiomenorrhea, delayed men-arche B. Amenorrhea, disordered eating, osteopo-rosis C. Disordered eating, iron deficiency, insom-nia D. Iliotibial band syndrome, decreased Q angle, ACL injury E. Stress fractures, patellofemoral pain, amenorrhea

B. Amenorrhea, disordered eating, osteopo-rosis

When completing a baseline posture assessment, an athlete was found to have a leg length discrepancy. Which of the following measurements will confirm this finding? A. Anterior inferior iliac spine to lateral malleolus B. Anterior superior iliac spine to medial malleolus C. Iliac crest to lateral malleolus D. Ischial tuberosity to calcaneal tubercle E. Umbilicus to medial malleolus

B. Anterior superior iliac spine to medial malleolus

Patient presents with itchy, scaling red rash on toes and webspaces. Which OTC medication is MOST appropriate? A. Antibiotic cream B. Anti fungal cream C. Oral anti histamine D. Oral NSAID E. Topical analgesic

B. Anti fungal cream

59. A patient sustained a blunt trauma to the face. Which of the following signs and symptoms are associated with a Le Fort fracture? Choose all that apply A. Bleeding around the lower teeth B. Bleeding around the upper teeth C. Inability to hear on the involved side D. Nasal bleeding E. Observation of a flattened cheek F. Unequal pupils

B. Bleeding around the upper teeth D. Nasal bleeding E. Observation of a flattened cheek

Which of the following supplements are most appropriate to prevent decreased bone mineralizations? A. calcium and niacin B. Calcium and vitamin D C. niacin and vitamin c D. vitamin C and vitamin E E. vitamin D and vitamin E

B. Calcium and vitamin D

17. The women's collegiate basketball coach approached the ATC with concerns about one of his athletes. The athletic trainer decides to schedule a meeting with the athlete to discuss the coach's concerns. The discussion reveals the athlete has extreme fatigue, weight loss, bradycardia, and light headedness when she stands up from seated or supine position. She states she cannot concentrate in class and does not feel well in general. She is having trouble sleeping at night and staying awake during her classes. She works out on her own after practices to earn a starting position on the team. The athlete is prescribed an antidepressant medication. Which of the following are common complications associate with antidepressant medications? A. Bradycardia B. Cardiac arrythmias C. Constipation D. Dilated pupils E. Nephritis F. Tachycardia

B. Cardiac arrhythmias C. Constipation F. Tachycardia

Which statement best describes Raynaud's phenomenon? A. cold exposure causes vasospasm in ma-jor veins of hands B. cold exposure causes vasospasm of digi-tal arteries C. cold exposure causes vasodilation of digi-tal arteries D. heat exposure causes vasodilation of dig-ital arteries E. heat exposure causes vasodilation of ma-jor veins in hand

B. Cold exposure causes vasospasm of digital arteries

Which type of fracture consists of three or more fractures at the site of injury? A. Avulsion B. Comminuted C. Greenstick D. Oblique E. Spiral

B. Comminuted

Which of the following devices is MOST appropriate to use on a tennis player diagnosed with lateral epicondylitis? A. Checkrein brace B. Counterforce forearm brace C. Elastic Bandage D. Elbow hinge brace E. neoprene forearm sleeve

B. Counterforce forearm brace

What is primary mechanism of action used by NSAIDs to decrease inflammation? A. Decrease acetylsalicylic acid production B. Decrease prostaglandin production C. Decrease sensititivty of chemoreceptors D. Decrease leukotriene production E. Decrease histamine production

B. Decrease prostaglandin production

The women's collegiate basketball coach approached the ATC with concerns about one of his athletes. The athletic trainer decides to schedule a meeting with the athlete to discuss the coach's concerns. The discussion reveals the athlete has extreme fatigue, weight loss, bradycardia, and light headedness when she stands up from seated or supine position. She states she cannot concentrate in class and does not feel well in general. She is having trouble sleeping at night and staying awake during her classes. She works out on her own after practices to earn a starting position on the team. Based on this information, what conditions would the athletic trainer suspect? Choose all that apply A. Athlete's position on team B. Depression C. Eating Disorder D. Loss of scholarship E. Mononucleosis F. Thyroid dysfunction

B. Depression C. Eating Disorder F. Thyroid Dysfunction

Which of the following signs and symptoms of anaphylactic shock would indicate administration of an epinephrine injection as prescribed for that athlete? Choose all that apply A. Diaphoretic B. Difficulty breathing C. Difficulty swallowing D. Lightheaded E. Pulse oxometer reading of 98% F. Swollen tongue

B. Difficulty breathing C. Difficulty swallowing F. Swollen tongue

Which of the following design techniques pro-tects against electric shock in hydrotherapy area? A. Electrical outlets should be located 18 inches above all floors B. Electrical outlets for whirlpools should be connected to GFCI C. Electrical outlets should be limited to three devices plugged in and operating D. Electrical on-off switches should be locat-ed within six inches of whirlpools

B. Electrical outlets for whirlpools should be connected to GFCI

What condition would disqualify an athlete from participation in a contact/collision sport? A. Absent or undescended testicle B. Enlarged spleen C. Inguinal hernia D. Sickle cell trait

B. Enlarged spleen

The women's collegiate basketball coach approached the ATC with concerns about one of his athletes. The athletic trainer decides to schedule a meeting with the athlete to discuss the coach's concerns. The discussion reveals the athlete has extreme fatigue, weight loss, bradycardia, and light headedness when she stands up from seated or supine position. She states she cannot concentrate in class and does not feel well in general. She is having trouble sleeping at night and staying awake during her classes. She works out on her own after practices to earn a starting position on the team. The athlete is returning to the athletic trainer for continued care and monitoring. Which of the following responsibilities is the athletic trainer PRIMARILY responsible for regarding the management of his athlete's care? Choose all that apply A. Ensure caregivers are aware of the treatment plan B. Ensure insurance and financial concerns are addressed C. Ensure the athlete is eating regular meals by providing them for the athlete D. Monitor compliance with the treatment plan by maintaining records of scheduled appoint-ments E. Return the player to their starting line up position F. Refill prescription orders

B. Ensure insurance and financial concerns are addressed

During large track meet, ath collapses with heat-related illness. When activating EMS, what essential information should the ATC provide to dispatcher? A. Age of participant B. Exact location of participant C. Full name of participant as found on medical history form D. Participant's insurance info E. Temperature and relative humidity at venue site

B. Exact location of participant

A deficiency in vitamin K could manifest into which of the following conditions? A. Anemia B. Excessive bruising C. Frequent Fractures D. Gastroenteritis E. Hypglycemia

B. Excessive bruising

Scenario: 28 y/o female ice hockey ath has horizontal laceration to left mid-anterior fore-arm. 2 teammates control bleeding and are covered in significant amount of blood. Eval of ath reveals 7cm wide 2cm deep laceration, BP 98/54 mmHg, pulse 115 bpm, rapid and shallow 24 breaths per minute, pale skin, conscious by lethargic. Question: Given location of the athlete's laceration, which of the following structures could be compromised by such an injury? A. Brachial Artery B. Flexor Digitorum Superficialis C. Median Nerve D. Pronator Teres E. Radial Artery F. Radial Nerve

B. Flexor Digitorum Superficialis C. Median Nerve E. Radial Artery

What are the MOST likely causes of tarsal tunnel syndrome? A. Extensor hallucis longus trauma B. Flexor hallucis longus trauma C. Fracture of calcaneous D. Fracture of distal fibula E. Lateral ankle sprain F. Medial ankle sprain

B. Flexor hallucis longus trauma C. Fracture of calcaneous F. Medial ankle sprain

While treating an athlete with back spasms the athletic trainer decides to perform massage. Which pain theory supports this decision? A. Central biasing theory B. Gate control theory C. Noxious pain modulation D. Peripheral pain modulation

B. Gate Control Theory

An athlete with Type 1 diabetes feels light-headed and dizzy. Which of the following is BEST to give him in order to prevent diabetic shock? A. Protein shake B. Glass of apple juice C. Glass of diet soda D. Saltine crackers E. Water

B. Glass of apple juice

Which statute can protect individuals from legal liability as long as they act without negligence and within the scope of their training? Choose one: A. Duty to act B. Good Samaritan law C. Medical immunity law D. Standard of care act

B. Good Samaritan law

When cleaning up dried blood, ATC is at GREATEST risk for contracting which one? A. Hepatitis A B. Hepatitis B C. Hepatitis C D. HIV E. TB

B. Hepatitis B

An 18 year old intercollegiate athlete enters that athletic training facility with complaints of not feeling well. Which of the following findings warrants a referral to a physician for diagnosis and treatment? Choose all that apply A. Dry cough B. High fever C. Malaise D. Nasal congestion E. Post nasal drip F. Purulent nasal discharge

B. High fever F. Purulent nasal discharge

In what position should an athlete's lower extremity be placed when applying a hip spica wrap for adductor strain? Select all that apply. A. Hip extension B. Hip flexion C. Hip internal rotation D. Hip external rotation E. Slight knee flexion F. Full knee flexion G. Knee extension

B. Hip flexion C. Hip Internal rotation E. Slight knee flexion

Which of the following statements represents BEST recommendation that an ATC could provide to tennis player suffering from lateral epicondylitis? A. Decrease racquet grip size and hold rac-quet tighter B. Hold racquet less tightly and limit number of backhand strokes C. Increase racquet grip size and limit the number of forehand strokes D. Stretch the shoulder musculature and decrease the racquet grip size E. Stretch the wrist flexors and strengthen the forearm pronators

B. Hold racquet less tightly and limit number of backhand strokes

The spinous process of the fourth lumbar vertebra is located at approximately the same level as which bony landmark? A. Greater trochanter B. Iliac crests C. Ischial tuberosity D. Median sacral crests E. PSIS

B. Iliac Crests

Which of the following employment interview situations is illegal? A. Interviewer asks for additional references from interviewee B. Interviewer asks about the interviewees marital status C. Interviewer asks about personal weaknesses D. Interviewee asks about medical benefits E. Interviewee asks about company's maternity leave

B. Interviewer asks about the interviewees marital status

A winging scapula may indicate an injury to which nerve? A. Axillary B. Long Thoracic C. Radial D. Musculocutaneous

B. Long Thoracic

Which of the following impedes thermal effects of ultrasound? A. Applying US with area stretched B. Mixing petroleum based sports creams with US gel C. Using a crystal with low beam non-uniformity ratio D. Using hot pack prior to US E. Using treatment time of less than 3 minutes

B. Mixing petroleum based sports creams with US gel

Which of the following HIPAA rules should athletic trainer's abide by when releasing health information to another health care entity? Choose all that apply A. Ensure the release of all detailed health infor-mation B. Obtain consent for treatment C. Obtain pertinent health history from immediate family members D. Release information to any interested parties E. Release only necessary information F. Safeguard patient information

B. Obtain consent for treatment E. Release only necessary information F. Safeguard patient information

After caring for an athlete with epistaxis, how should the athletic trainer handle the towel with dried blood spots on it? Choose all that apply A. Hand wash separately in hot water B. Separate towel from other laundry C. Spray with disinfectant and place in laundry bin D. Throw towel away in trash receptacle E. Transport in biohazard container F. Wash in hot water with bleach

B. Separate towel from other laundry E. Transport in biohazard container F. Wash in hot water with bleach

What is the recommended time period, in years, that every NOCSAE approved helmet should be returned to the manufacturer for recertification/reconditioning? A. one B. two C. three D. four E. five

B. Two

Which of the following conditions is BEST de-scribed as the heart's inability to pump blood due to rapid, but unorganizaed contractions? A. Ventricular tachycardia B. Ventricular fibrillation C. Bradycardia D. Atrial flutter E. Asystole

B. Ventricular fibrillation

Which of the following conditions would be treated by these medications? Match the options from the toolbar on the left to the targets listed in the table to the right. Each option can be used once. Medication: Condition: Anti- bacterial Bronchitis Anti- fungal Carbuncle Anti- inflammatory Eczema Anti- pruritic Herpes Simplex I Anti- tussive Poison Ivy Anti- viral Tinea Cruris

BRONCHITIS: ANTI-TUSSIVE CARBUNCLE: ANTI-BACTERIAL ECZEMA: ANTI-INFLAMMATORY HERPES SIMPLEX I: ANTI-VIRAL POISON IVY: ANTI-PRURITIC TINEA CRURIS: ANTI-FUNGAL

Which of the following describes the MOST common site of Achilles tendon ruptures? A. At the musculotendinosus junction B. At the proximal attachment of the tendon C. 1-2 inches proximal to the calcaneal attachment D. Parallel to the fibers of the tendon

C. 1-2 inches proximal to the calcaneal attachment

What is the minimum amount of time in minutes time should pass after the last sound of thunder or the last lightening strike before resuming play? A. 10 min B. 15 min C. 30 min D. 45 min

C. 30 min

Which statement BEST describes Marfan syndrome? A. A hereditary condition involving blood and other bodily fluids B. A hereditary condition involving inflammation of the joint C. A hereditary condition involving the connective tissue, bones, muscles, and ligaments D. A hereditary condition involving the scarring of myelin sheath

C. A hereditary condition involving the connective tissue, bones, muscles, and ligaments

Which of the following is the appropriate management for a Type 1 diabetic athlete with a blood glucose reading of 320 mg/dl? Choose all that apply A. Activate EMS B. Administer 8 oz of orange juice C. Administer insulin bolus D. Assess for ketones in urine E. Prepare a glucagon injection F. Remove from participation

C. Administer insulin bolus D. Assess for ketones in urine F. Remove from participation

Which of the following individuals should be involved in the development of an emergency action plan? Choose all that apply A. Athletes B. Athletic training students C. Athletic trainers D. Emergency care service providers E. Parents F. Physicians

C. Athletic trainers D. Emergency care service providers F. Physicians

What governing body regulates the fraudulent reporting of continuing education units by athletic trainers? A. American Alliance of Health, PE, Rec, Dance B. American Medical Association C. BOC, Inc D. CAATE E. NATA

C. BOC, Inc

When developing a treatment protocol policy using EBP, which is MOST appropriate? A. Consult experts in field B. Conduct a thorough internet search C. Conduct a thorough review of literature D. Discuss the protocol with other profes-sionals E. Examine NATA position statements

C. Conduct a thorough review of literature

A 15 y/o female ath reports suffering from pain and moderate cramping in R side of ab-domen. ATC eval reveals athlete is nauseat-ed, low grade fever, demonstrated marked tenderness between ASIS and umbilicus. Following initial eval, what is MOST appropriate action for AT to take? SELECT ALL THAT APPLY A. Administer an anti-inflammatory B. Administory antacid C. Contact athlete's parents D. Hold athlete out of physical activities and observe her for other symptoms E. Refer to ED

C. Contact athlete's parents E. Refer to ED

A majority of rotator cuff injuries develop during which phase of overhead throwing? A. Acceleration B. Cocking C. Deceleration D. Windup

C. Declaration

Scenario: 28 y/o female ice hockey ath has horizontal laceration to left mid-anterior forearm. 2 teammates control bleeding and are covered in significant amount of blood. Eval of ath reveals 7cm wide 2cm deep laceration, BP 98/54 mmHg, pulse 115 bpm, rapid and shallow 24 breaths per minute, pale skin, conscious by lethargic. Question: Ath is transported to ED where they find ath is positive for HBV. Which are methods of transmission for HBV? A. Contaminated Food B. Contaminated water C. Direct contact with blood D. Direct contact with perspiration E. Oral/fecal contact F. Unprotected sexual contact

C. Direct contact with blood F. Unprotected sexual contact

When writing a position description for an athletic training job, what should be included in the components of the job specification? Choose all that apply A. Applicant's health history B. Cognitive function demands C. Education requirements D. Physical demands of the job E. Required certifications F. Volunteer community service duties

C. Education requirements D. Physical demands of job E. Required certification

In response to an athlete that was tackled during a football game, the athletic trainer assessed his condition as a solar plexus injury. What is the INITIAL on field management? Choose all that apply A. Activate EMS B. Apply ice bag to the area C. Encourage the athlete to relax and regulate his breathing D. Initiate CPR and set up the AED for further instructions E. Loosen restricting equipment and clothing F. Treat for shock by placing athlete supine with legs elevated

C. Encourage the athlete to relax and regulate his breathing E. Loosen restricting equipment and clothing

Which type of third-party provides practitioners fixed fees for services, limits its members' choices to pre-approved practitioners, and requires the use of a primary care physician? A. Point of service (POS) B. Preferred Provider Organization (PPO) C. Health Maintenance Organization (HMO) D. Fee for service (FOS) E. Exclusive Provider Organization (EPO)

C. Health Maintenance Organization (HMO)

An athlete reports to the athletic training facility complaining of light-headedness, dizziness, nausea, and referred pain to the inferior angle of the right scapula. What condition should be suspected at this time? A. Abdominal hernia B. Appendicitis C. Hepatomegaly D. Ovarian cyst E. Splenomegaly

C. Hepatomegaly

Which of the following conditions is defined as an accumulation of blood in the anterior chamber of the eye following blunt trauma? A. corneal abrasion B. Hordeolum C. Hyphema D. Retinal detachment E. Ruptured globe

C. Hyphema

As a part of the rehab process, an athletes learns to visualize the desired outcomes. What psychological intervention strategy is being used? A. Cognitive restructuring B. Goal setting C. Imagery D. Positive self-talk E. Relaxation

C. Imagery

Which of the following is MOST beneficial for athletes suffering from the common cold? A. Decrease calorie intake B. Decrease fluid intake C. Increase fluid intake D. Increase vitamin C intake E. Use herbal supplements

C. Increase fluid intake

The women's collegiate basketball coach approached the ATC with concerns about one of his athletes. The athletic trainer decides to schedule a meeting with the athlete to discuss the coach's concerns. The discussion reveals the athlete has extreme fatigue, weight loss, bradycardia, and light headedness when she stands up from seated or supine position. She states she cannot concentrate in class and does not feel well in general. She is having trouble sleeping at night and staying awake during her classes. She works out on her own after practices to earn a starting position on the team. The athletic trainer suspects an eating disorder based on information shared during further consultation with the athlete. Which of the following methods are PRIMARILY recommended to determine whether or not this athlete has an eating disorder? A. interview a parent B. Interview a teammate C. Interview the athlete D. Interview the coach E. HIPAA form F. PPE G. Self-evaluation

C. Interview the athlete F. PPE G. Self-evaluation

Which BEST describes nystagmus? A. Eye swollen shut B. Inability of the pupil to accommodate to light C. Involuntary lateral movement of eye D. Rapid decrease in visual acuity E. Unequal pupil size

C. Involuntary lateral movement of eye

Athlete has been diagnosed with lateral epicondylitis. As part of treatment plan, ATC has decided to apply a counterforce brace. Which of the following options is the CORRECT placement for counterforce brace? A. at mid forearm B. directly on lateral epicondyle C. just distal to lateral epicondyle D. on pronator teres E. Proximal to lateral epicondyle

C. Just distal to lateral epicondyle

Which of the following BEST determines the scope of practice for a clinical AT? A. Standing orders from physician B. Employment policies of facility C. Laws of state D. BOC Standards of Professional Practice E. BOC Role Delineation Study/Practice Analysis

C. Laws of state

Which of the following tests could an athletic trainer use to confirm a suspected bicep brachii tendon rupture? A. Yergason's B. Obrien's C. Ludington's D. Hawkins-Kennedy E. Adson's

C. Ludington's

In which direction should stirrup strips be applied to a patient with laxity in the calcaneofibular ligament? A. Anterior to posterior B. Lateral to medial C. Medial to lateral D. Posterior to anterior E. Spiral lateral to medial

C. Medial to Lateral

An athlete reports having periodic numbness and intermittent pain in the 3rd and 4th toes of both feet when wearing dress shoes. Based on this information, what condition should the ATC suspect? A. Jones fracture B. Metatarsalgia C. Morton's neuroma D. Sesamoiditis E. Tailor's bunion

C. Morton's Neuroma

How would you describe isometric exercise to athlete? A. Variable resistance applied over a joint range of motion B. Constant velocity through limited range of motion C. Muscle generates force by no joint movement D. Muscle generates force in only one direc-tion of movement E. Moving joint performs a range of motion with a set of resistance in both directions

C. Muscle generates force by no joint movement

What is the primary purpose of the overload principle? A. Muscular endurance is improved by increasing amount of weight lifted B. Muscular endurance is improved by increasing amount of weight lifted over one month C. Muscle strength is improved by the muscle being forced to harder than it is accustomed to D. Muscular strength is improved by the same muscle group being lifted daily E. Muscular strength is improved by visualization

C. Muscle strength is improved by the muscle being forced to harder than it is accustomed to

An athlete reports to the athletic trainer complaining of tingling and numbness on the lateral surface of the forearm. Which peripheral nerve is MOST likely involved with this athlete's complaint? A. Medial cutaneous B. Medial C. Musculocutaneous D. Radial E. Ulnar

C. Musculocutaneous

What are the nutritional principles that aid in recovery from acute musculoskeletal injury? Choose all that apply A. Cabbage soup diet B. Clear liquid diet C. Nutrient dense, varied diet to ensure adequate uptake of vitamins and minerals D. Sufficient hydration E. Sufficient protein intake F. Supplements and water

C. Nutrient dense, varied diet to ensure adequate uptake of vitamins and minerals D. Sufficient hydration E. Sufficient protein intake

A patient presents with the signs and symp-toms of L4-L5 disc herniation. Which of the following neurological assessments will aid the ATC in confirmation of diagnosis? SELECT ALL THAT APPLY. A. Deficit with Achilles reflex B. Deficit with patella reflex C. Parasthesia at dorsum of foot D. Parasthesia at medial aspect of tibia E. Weakness during big toe extension F. Weakness during dorsiflexion

C. Parasthesia at dorm of foot E. Weakness during big toe extension

Which is MOST likely to result in medial tibial stress syndrome? A. pronation creating tension on proximal anterior tibialis attachment B. Pronation creating tension on the distal anterior tibialis attachment C. Pronation creating tension on proximal posterior tibialis attaachment D. Supination causing tension at the proxi-mal flexor digitorum attachment E. Supination causing tension on the proxi-mal flexor hallicus longus attachment

C. Pronation creating tension on proximal posterior tibialis attaachment

Which of the following are appropriate management strategies for a tooth avulsion? Select all that apply. A. Place in gauze on ice B. Place in dry gauze C. Refer to dentist immediately D. Replace into the socket E. Rice with bottled water F. Rinse with saline water

C. Refer to dentist immediately D. Replace into the socket F. Rinse with saline water

What type of electrical current is recommended to BEST prevent muscle atrophy? A. IFC B. Microcurrent C. Russian current D. Sensory level high volt pulse current

C. Russian Current

An athlete reports to ATR suffering from chronic injury to long thoracic nerve. To which muscle should rehab exercises be directed? A. Levator scapula B. Rhomboids C. Serratus anterior D. Supraspinatus E. Teres minor

C. Serratus anterior

A high school basketball athlete is recovering from an anterior glenohumeral dislocation; she feels she is ready to return to full practice and games. What are the return to play criteria? Choose all that apply A. A shoulder brace must be worn for practice and game B. Kinesiotape applied prior to practice and games C. Shoulder strength should be nearly normal D. The shoulder should have full range of motion and be pain free E. The shoulder should have limited range of motion and be pain free F. Throwing and catching activities should not produce pain

C. Shoulder strength should be nearly normal D. The shoulder should have full range of motion and be pain free F. Throwing and catching activities should not produce pain

What landmark appears as a depression in the forefoot and can be palpated over anterolateral talus? A. Cuboid B. Peroneal tubercle C. Sinus tarsi D. Styloid process of 5th met E. Talar dome

C. Sinus tarsi

Scenario: 28 y/o female ice hockey ath has horizontal laceration to left mid-anterior forearm. 2 teammates control bleeding and are covered in significant amount of blood. Eval of ath reveals 7cm wide 2cm deep laceration, BP 98/54 mmHg, pulse 115 bpm, rapid and shallow 24 breaths per minute, pale skin, conscious by lethargic. Question: When arriving on the ice to treat the injured athlete, the athletic trainer activates EMS and applies a pressure bandage to control bleeding. Which actions should AT take until arrival of EMS? A. apply tourniquet B. clean blood off the ice C. elevate affected arm D. elevate athletes legs E. instruct exposed teammates to clean blood off of themselves F. Instruct exposed teammates to remove their bloody equipment G. maintain athlete's body temperature

C. elevate affected arm D. elevate athletes legs G. maintain athlete's body temperature

When measuring ankle DF ROM, at which lo-cation should moveable arm of goniometer be aligned? A. first met B. second met C. fifth met D. fibula E. tibia

C. fifth met

A warm whirlpool immersion treatment should not exceed what temperature to avoid scaling? Choose one: A. 100 degrees F(38 degrees C) B. 105 degrees F (41 degrees C) C. 110 degrees F (43 degrees C) D. 115 degrees f (46 degrees C) E. 120 degrees F (49 degrees C)

D. 115 degrees f (46 degrees C)

What is accepted method of immediate treatment for acute anterior compartment syndrome prior to referring to a physician? A. Apply moist heat and stretch limb B. Apply ice and compression and elevate limb C. Apply compression with elastic bandage D. Apply ice and elevate the limb E. Apply a lower leg air splint

D. Apply ice and elevate the limb

An east coast softball team is traveling to the west coast for a playoff series. What are the strategies to reduce the negative effects of circadian dysrhythmia? Choose all that apply A. Adjust practice schedule to earlier in the day B. Avoid caaffeine in coffee, soda, and tea C. Avoid sunlight D. Awake and sleep one hour later for each time zone crossed E. Drink plenty of fluids F. Get less sleep the day prior to departure G. Reset watches to local time zone prior to departure

D. Awake and sleep one hour later for each time zone crossed E. Drink plenty of fluids G. Reset watches to local time zone prior to departure

When working in industrial setting, which instructions should be given to an individual who lifts heavy boxes? A. Brace abdominal muscles and twist hips and low back B. Brace abdominal muscles, bend forward slightly, and use your arms to lift the ob-ject C. Squat with weight on your toes and use your hamstrings and quads to lift object D. Brace your abdominal muscles, keep a neutral spine, and use your hip and knee extensors to lift object

D. Brace your abdominal muscles, keep a neutral spine, and use your hip and knee extensors to lift object

Which of the following a comobidity related to disordered eating that has been shown to cause sudden death? A. amenorrhea B. anemia C. dysmenorrhea D. electrolyte abnormalities E. osteoporosis

D. Electrolyte abnormalities

A 25-year-old athlete persons complaining of nausea and a general feeling of malaise. The athlete reports vomitting the previous night. This athlete has a history of antacid-relieved stomach maladies. To which of the following allied healthcare professionals should the athletic trainer refer to this athlete? A. Otolaryngologist B. Psychologist C. Internist D. Gastroenterologist E. Dietician

D. Gastroenterologist

Three days after sustaining a blow to the head in a middle school soccer match, the athlete reports to the athletic trainer that she has been experiencing mild headaches with increased activity. What is the MOST appropriate action for the athletic trainer to take? A. Allow the athlete to complete a restricted prac-tice and re-evaluate following practice B. Check the athlete's vital signs and clear her for participation if within normal limits C. Closely monitor the athlete at practice and no-tify her parents at its conclusion D. Hold the athlete from practice and notify her parents E. Perform functional testing to check the ath-lete's status

D. Hold the athlete from practice and notify her parents

According to NATA position statement, which of the following is MOST appropriate initial response for an exertional heat stroke? A. Apply ice bags to major arteries B. Douse the patient with cool water C. Activate EMS D. Immerse in ice water E. Place the patient in the shade

D. Immerse in ice water

Which of the following pathologies is MOST likely to refer to pain to the patient's lower back? A. Appedicitis B. Inguinal hernia C. Hiatal hernia D. Kidney stones E. Gall stones

D. Kidney stones

Which of the following is the MOST common mechanism of injury for a cervical nerve root contusion? A. Cervical hyperextension with a cervical com-pression B. Cervical hyperextension with a rotational force C. Forward cervical flexion with an axial load D. Lateral cervical flexion with compression

D. Lateral cervical flexion with compression

Which of the following systems regulates homeostasis during rigorous exercise? A. Cardiovascular system B. Digestive system C. Lymphatic system D. Nervous system E. Respiratory system

D. Nervous system

A 13 year old baseball athlete complains of pain and locking in the lateral aspect of the elbow. Observation reveals loss of ROM and increased swelling. What condition should the athletic trainer suspect at this time? A. Cubital tunnel syndrome B. Lateral epicondylitis C. Little league elbow D. Osteochondritis dissecans E. Pronator teres syndrome

D. Osteochondritis dissecans

An athlete reports feeling irritable and fa-tigued on a regular basis. ATC notices that this athlete often stays on the field after practice to run extra sprints and also puts in extra time in the weight room. Ath has pre-sented with multiple overuse injuries during last two months and his overall performance has decreased. Based on the information, what condition should ATC suspect? A. Anorexia nervosa B. Depression C. OCD D. Overtraining syndrome E. SAD

D. Overtraining Syndrome

What is the primary purpose for utilizing taping techniques following a sprain of a joint? A. Assist acute inflammation B. Decrease skin temperature C. Increase joint ROM D. Provide support and reduce stress to joint musculature and soft tissue E. Strengthen musculature around joint

D. Provide support and reduce stress to joint musculature and soft tissue

Athlete suffering from tinea cruris would MOST likely present with which of the follow-ing? A. Erythematous and scaly area on skin B. Hair loss and scaly areas on head C. Thicking and opaque coloring of nail bed D. Scaly and pruritic rash on groin

D. Scaly and pruritic rash on groin

A wide receiver fell on his elbow and left side during game. He now reports left shoulder pain and nausea with rigidity and rebound tenderness in L abdominal quadrant. Which of the following concerns should the ATC suspect? A. AC joint sprain B. Appendix injury C. Rib fracture D. Spleen injury

D. Spleen Injury

Field hockey player reports being hit by a shot in lateral chest wall. She complains of pain at the site of impact and dyspnea. ATC notes that she is becoming cyanotic and ob-serves tracheal deviation away from injured side. Based on this info, what condition should the ATC suspect? A. Clavicular fracture B. Myocardial infarction C. Rib fracture D. Tension pneumothorax E. Traumatic asphyxia

D. Tension Pneumothorax

Which of the following are true concerning Salter-Harris fractures? Choose all that apply A. They are associate with overuse injuries B. They are most common in female athletes C. They involve the diaphysis of the bone D. They involve the epiphyseal plate E. They occur in adolescent athlete

D. They involve the epiphyseal plate E. They occur in adolescent athlete

What bone articulates with the proximal end of the first metacarpal? A. Capitate B. Lunate C. Navicular D. Trapezium E. Triquetrum

D. Trapezium

What combination of factors would MOST likely predispose an athlete to patellofemoral pain? A. femoral retroversion, tight hamstrings, genu varum B. Tight qudas, coxa vara, Q angle less than 15 degrees C. Weak quads, pes cavus, tight iliopsoas D. Weak quads, genu valgum, Q angle over 18 degrees E. Weak hamstrings, genu valgum, Q angle greater than 13 degrees

D. Weak quads, genu valgum, Q angle over 18 degrees

An athlete recently sustained a subluxated patella. She has been cleared by her physi-cian to return to play, but she must wear a brace. Which of the following braces would be MOST beneficial for this athlete? A. custom made functional knee brace B. double hinged rigid knee brace C. neoprene sleeve with medial hinge D. neoprene sleeve with patella centralizing brace E. prophylactic lateral hinged knee brace

D. neoprene sleeve with patella centralizing brace

The patient's prescription states: "take 1 tablet bid". How often do they need to take medicine? A. as needed B. one time at breakfast C. one time at dinner D. two times a day E. three times a day

D. two times a day

When disinfection contaminated surfaces, what concentration of bleach solution is recommended by OSHA? A. 1:2 B. 1:3 C. 1:5 D. 1:7 E. 1:10

E. 1:10

Which of the following is the MOST appropriate medication treatment for herpes simplex? A. Antibiotic B. Antifungal C. Aninflammatory D. Antitussive E. Antiviral

E. Antiviral

What recommendation should be given to an athlete who has recently diagnosed with asthma? A. Encourage athlete to receive an annual EKG test B. Encourage athlete to receive annual chest xray C. Encourage athlete to monitor blood pressure daily D. Encourage athlete to discontinue all ath-letic activity E. Encourage athlete to bring rescue MDI to all athletic activities

E. Encourage athlete to bring rescue MDI to all athletic activities

Which part of the bone is damaged in a diagnosed type 2 saltar harris fracture? A. Articular surface B. Condyles C. Diaphysis D. Epicondyles E. Epiphyseal plate

E. Epiphyseal plate

Bradycardia that develops secondary to regular aerobic exercise is the result of what specific physiological adaptation? A. decreased respiratory rate B. decreased stroke volume C. increased heart rate D. increased respiratory rate E. increased stroke volume

E. Increased stroke volume

Which of the following is an epiphyseal condition that causes tissue necrosis and potential deformation to the head of the femur? A. Sever's disease B. Scheurmann's disease C. Os trigonum disease D. Osgood-Schlatter disease E. Legg-Calve-Perthes Syndrome

E. Legg-Calve-Perthes Syndrome

What bony structure should the athletic trainer palpate to assess for plantar fasciitis? A. Talus B. Sustentaculum tali C. Peroneal tubercle D. Navicular tubercle E. Medial calcaneal tubercle

E. Medial calcanea tubercle

Which of the following agencies publishes guidelines for the reconditioning and maintenance of helmets for football, lacrosse, baseball? A. American Society for Testing and Materi-als B. Canadian Standards Association C. Consumer Product Safety Commission D. International Organization for Standardiza-tion E. National Operating Committee on Stand-ards for Athletic Equipment

E. National Operating Committee on Stand-ards for Athletic Equipment

Which of the following pathologies is a contra-indiction to mechanical traction? A. Acute disc herniation B. Degenerative joint disease C. Joint hypomobility D. Muscle spasm E. Osteoporosis

E. Osteoporosis

During wrestling practice an athlete is hit in the groin. The athletic trainer's evaluation reveals a testicular contusion. Which of the following is the BEST action to take at this time? A. Place athlete in hook lying position B. Place athlete in quadruped position C. Place athlete in recumbent position D. Place athlete in sitting position with legs ex-tended E. Place athlete in supine position with knee flexed toward chest

E. Place athlete in supine position with knee flexed toward chest

What term describes an athletic trainer's ability to influence others in their organization? A. Authority B. Leadership C. Management D. Monopoly E. Power

E. Power

After treating athlete for TOS for 3 days, ath returns ℅ decreased ball velocity and tired and heavy arm. ATC reevals and finds: swell-ing in pitching arm, normal symmetrical radial pulse, cool skin temp, 5/5 wrist and elbow strength. What action should ATC take at this time? A. Apply electrical stim for edema reduction B. Apply compression sleeve and elevate C. Continue strengthening, stretching, and ice D. Perform milking massage E. Refer for further evaluation

E. Refer for further evaluation

What should be suspected in any patient that collapses and is unresponsive? A. Cervical spine injury B. Diabetes mellitus C. Exertion heat stroke D. Exertional sickling E. Sudden cardiac arrest

E. Sudden cardiac arrest

The women's collegiate basketball coach approached the ATC with concerns about one of his athletes. The athletic trainer decides to schedule a meeting with the athlete to discuss the coach's concerns. The discussion reveals the athlete has extreme fatigue, weight loss, bradycardia, and light headedness when she stands up from seated or supine position. She states she cannot concentrate in class and does not feel well in general. She is having trouble sleeping at night and staying awake during her classes. She works out on her own after practices to earn a starting position on the team. The athletic trainer refers the athlete for an initial consultation. Which of the following health care professionals should she see first? A. Clinical psychologist B. Nurse practitioner C. Registered dietician D. Sports psychologist E. Team physician

E. Team Physician

A high school athlete wishes to join the track and field team as a pole vaulter. Due to the poten-tial risk for catastrophic injury, the athlete is re-quired to purchase additional medical coverage and the athlete's guardian will need to sign an assumption of risk waiver per school district poli-cy. Which form of risk reduction is used by the school district? A. Avoidance B. Records Management C. Reduction D. Retention E. Transference

E. Transference

At which point in the injury response cycle will the application of ice be MOST beneficial in reducing secondary hypoxia? A. Diapedesis and margination B. Pain and spasm C. Rehabilitation D. Repair phase E. Vascular and tissue damage

E. Vascular and tissue damage

Scenario: 28 y/o female ice hockey ath has horizontal laceration to left mid-anterior fore-arm. 2 teammates control bleeding and are covered in significant amount of blood. Eval of ath reveals 7cm wide 2cm deep laceration, BP 98/54 mmHg, pulse 115 bpm, rapid and shallow 24 breaths per minute, pale skin, conscious by lethargic. Question: 3 days post surgery to repair structures athlete complains of pressure and pain in forearm, and there is decreased finger extension. Which of the following possible differential diagnosis should athletic trainer consider? A. Ape hand B. Dupuytren's contracture C. Pronator teres syndrome D. Bishop's hand E. Volkman's ischemic contracture

E. Volkman's Ischemic Contracture

Which of the following BEST defines bone and soft tissue's response to stress? A. Archimede's principle B. Frank-starling law C. Inverse cosine law D. Law of Grotthus-Draper E. Wolff's law

E. Wolff's Law

A construction foreman is injured on the job site. What PRIMARY insurance coverage would cover this individual? A. Health Maintenance Organization B. Medicaid C. Medicare D. Liability insurance E. Worker's Compensation

E. Worker's Compensation

Which of the following is the most appropraite progressive functional sequence for the up-per extremity in a volleyball player? A. setting, forearm passing, hitting, over-head serving B. Overhand serving, forearm passing, set-ting, hitting C. Hitting, underhand serving, forearm pass-ing, setting D. Forearm passing, underhand serving, hit-ting, setting E. forearm passing, setting, overhead serv-ing, hitting

E. forearm passing, setting, overhead serv-ing, hitting

What lifestyle modification is recommended for a patient with hypertension? A. reduce aerobic exercise B. reduce intake of dairy C. reduce intake of all unprocessed foods D. reduce intake of potassium E. reduce intake of sodium

E. reduce intake of sodium

Which of the following physiological responses would occur in which phase of the healing process? Match options from the toolbar on the left to targets listed in the table to the right. Each option can be used more than once. Phase Physiological Response Fibroblastic - Collagen Fiber Production Inflammatory - Collagen Fiber Realignment Maturation - Extracellular Matrix -Histamine Response -Phagocytosis -Platelet Plug Formation -Restoration of Tissue Integrity -Scar Formation

FIBROBLASTIC REPAIR PHASE A. COLLAGEN FIBER PRODUCTION B. EXTRACELLULAR MATRIX C. HISTAMINE RESPONSE D. SCAR FORMATION INFLAMMATORY PHASE A. PHAGOCYTOSIS B. PLATELET PLUG FORMATION MATURATION PHASE A. COLLAGEN FIBER REALIGNMENT B. RESTORATION OF TISSUE INTEGRITY

In which stage of healing would it be most beneficial to use the following modalities? Match the options from the toolbar on the left to the targets listed in the table to the right. Each option can be used more than once. Phase: Modalities: Inflammation Phase Cold Modalities Maturation Phase Continuous Ultrasound Heat Modalities Intermittent Compression Massage Pulsed Ultrasound

INFLAMMATION PHASE A. COLD MODALITIES B. INTERMITTENT COMPRESSION C. PULSED ULTRASOUND MATURATION PHASE A. CONTINUOUS ULTRASOUND B. HEAT MODALITIES C. MASSAGE

A basketball player suffers a Grade II inversion ankle sprain. Of the following, which is the most appropriate progression of rehabilitation exercises? Place the exercises from the toolbar on the left in the correct sequence listed in the table to the right. Not every exercise will be used. Only one exercise should be placed in each target. Please note: Place the selected tool from the toolbar directly into the target area. Ensure that the tool snaps into the target area. To remove a tool from a target area, click the X in the upper right corner of the tool. Only one tool should be placed in each target area. Exercises: Cutting Isometrics Range of Motion Resistance Tubing Running Sequence Tables 1st 2nd 3rd 4th 5th

Isometrics (First) Range of Motion (Second) Resistance Tubing (Third) Running (fourth) Cutting (fifth)

Which of the following statements are characteristics of open and closed chain exercises? Match the options from the toolbar on top to the targets listed in the table on the bottom. Each option can be used more than once. Type of Exercise: Open Chain Closed Chain Statement: - Distal Segment moves in space - External stabilization usually required - Muscle Activation occurs in multiple muscular groups - Resistance is applied simultaeneously to multiple moving segments - Use of axial loading

OPEN CHAIN: - DISTAL SEGMENT MOVES IN SPACE - EXTERNAL STABILIZATION USUALLY REQUIRED CLOSED CHAIN: - MUSCLE ACTIVATION OCCURS IN MULTIPLE MUSCULAR GROUPS - USE OF AXIAL LOADING - RESISTANCE IS APPLIED SIMULTANEOUSLY TO MULTIPLE MOVING SEGMENTS

Which PPB category does the following list of patient information belong? Match the category with the appropriate piece of information. Match the categories from the toolbar on the left to the information listed in the table to the right. Each category can be used once. PPB Category: Information: Cardiovascular Screen Hamstring Flexibility Maturity Assessment Height,Weight, BP Medical History Orthopedic Screen Marfan Syndrome Previous Appendectomy Physical Examination Tanner's Stage 4

ORTHOPEDIC SCREEN: HAMSTRING FLEXIBILITY PHYSICAL EXAMINATION: HEIGHT, WEIGHT, BLOOD PRESSURE CARDIOVASCULAR SCREEN: MARFAN SYNDROME MEDICAL HISTORY: PREVIOUS APPENDECTOMY MATURITY ASSESSMENT: TANNER'S STAGE 4

Several athletes are being treated in the athletic training facility for a variety of injuries. Which of the following types of electrical stimulation currents are most appropriate to treat the following conditions? Match the type of electrical stimulation from the toolbar on the left to the condition listed in the table to the right. Each type can be used once. Please note: Place the selected tool from the toolbar directly into the target area. Ensure that the tool snaps into the target area. To remove a tool from a target area, click the X in the upper right corner of the tool. Only one tool should be placed in each target area. Type of Stimulation: Pulsed High-Volt Micro-current Medium Frequency Russian lnterferential Current Low Frequency TENS Condition: Muscle strengthening Wound Healing Muscle Re-education Active Pain Long-Term Chronic Pain

Pulsed High-Volt= Muscle Strengthening Micro-current= Wound healing Medium Frequency Russian = Muscle Re-education Low Frequency TENS = Long-term chronic pain lnterferential Current = Active Pain

Complete the soap note in order: Wrist strength 3/5 Possible R wrist sprain Athlete reports pain began two days prior increase R wrist strength to 5/5

S: Athlete reports pain began two days prior O: Wrist strength 3/5 A: Possible R wrist sprain P: Increase R wrist strength to 5/5

Aqua therapy is being utilized as a part of the rehabilitation program for a grade 2 MCL sprain of the knee. Identify the order of progression for the following exercises from the easiest to most difficult. Match the options from the toolbar on top to the targets listed in the table on the bottom. Each option can be used once. Exercise: Bicycle kicks in deep water Double leg hops in waist deep water Double leg lift in deep water Walking in neck deep water Stage Step 1: Step 2: Step 3: Step 4: Step 5:

STEP 1: DOUBLE LEG LIFTS IN DEEP WATER STEP 2: BICYCLE KICKS IN DEEP WATER STEP 3: WALKING IN NECK DEEP WATER STEP 4: STEP-UPS IN CHEST DEEP WATER STEP 5: DOUBLE LEG HOPS IN WAIST DEEP WATER

A hockey player reports to the athletic trainer complaining of right hip pain. The athlete indicates that the pain began approximately three weeks ago and is progressively increasing. The athlete has no prior history of hip injuries. The athletic trainer completes an initial evaluation of this athlete. Initial observations: • No atrophy or ecchymosis present • Right iliac crest and ASIS appear inferior • Pronated feet/ pes planus • Genu Varum • Apparent weight shift to the left side The athletic trainer's evaluation of this athlete reveals the following goniometric measurements of the hip. Which of the noted measurements are abnormal? *Choose all that apply: a) 10 degrees of extension b) 12 degrees of adduction c) 23 degrees of external rotation d) 38 degrees of internal rotation e) 47 degrees of abduction f) 129 degrees of flexion

a) 10 degrees of extension b) 12 degrees of adduction c) 23 degrees of external rotation

A hockey player reports to the athletic trainer complaining of right hip pain. The athlete indicates that the pain began approximately three weeks ago and is progressively increasing. The athlete has no prior history of hip injuries. The athletic trainer completes an initial evaluation of this athlete. Initial observations: • No atrophy or ecchymosis present • Right iliac crest and ASIS appear inferior • Pronated feet/ pes planus • Genu Varum • Apparent weight shift to the left side Based on the signs and symptoms identified during the evaluation, which of the following conditions should the athletic trainer suspect? Choose all that apply: a) Abductor muscle strain b) Adductor muscle strain c) Femoral neck stress fracture d) Hip flexor strain e) Lumbar disk herniation f) Trochanteric bursitis

a) Abductor muscle strain c) Femoral neck stress fracture e) Lumbar disk herniation

Which of the following signs, symptoms, and/or test results would indicate a diagnosis of thoracic outlet syndrome? Choose all that apply: a) Absent radial pulse in military brace position b) Absent radial pulse with active compress ion test c) Complaint of hot/burning fingers after pitching d) Complaint of loose feeling in shoulder while pitching e) Diminished radial pulse during a Speed's Test f) Diminished radial pulse during an Adson's Test g) Numbness/tingling in the upper extremity after pitching h) Pallor in the upper extremity after activity

a) Absent radial pulse in military brace position f) Diminished radial pulse during an Adson's Test g) Numbness/tingling in the upper extremity after pitching h) Pallor in the upper extremity after activity

Two athletes collide during a lacrosse match. One athlete continues playing while the other athlete is unconscious, but breathing with a palpable pulse. What is the most appropriate immediate action for the athletic trainer to take? Choose One: a) Activate EMS and monitor the athlete's vital signs b) Begin secondary survey to assess for further injuries c) Establish airway, breathing, and circulation d) Remove protective head gear and activate the emergency action plan e) Spine board the athlete for immediate transport to the emergency room

a) Activate EMS and monitor the athlete's vital signs

Which of the following are contraindications for iontophoresis? Choose all that apply: a) Analgesia b) Arterial disease c) Inflammation d) lschemia e) Medication sensitivity f) Open lesions

a) Analgesia e) Medication sensitivity f) Open lesions

Which of the following are major triggers of asthma? Choose all that apply: a) Animal dander b) Bee stings c) Ear infections d) Peanuts e) Pneumonia f) Pollens g) Stress or anxiety h) Tobacco smoke

a) Animal dander f) Pollens g) Stress or anxiety h) Tobacco smoke

Which of the following is the most likely cause of Commotio Cordis? Choose One: a) Blunt trauma to the chest b) Congenital abnormality c) Fluid accumulation around the heart d) Heart puncture by a rib e) Hypertrophic cardiomyopathy

a) Blunt trauma to the chest

To prevent dehydration, athletic trainers should encourage participants to ingest fluids to maintain their urine to which color? Choose One: a) Clear to light yellow b) Brown c) Dark yellow d) Yellow

a) Clear to light yellow

A properly fitted sports bra helps minimize stretching to which anatomical structure? Choose One: a) cooper's ligament b) ligament of Humphrey c) ligament of wrisberg d) nuchal ligament e) round ligament

a) Cooper's Ligament

Which of the following describes the signs and symptoms of heat stroke? Choose all that apply: a) Disorientation b) Red/dry skin c) Rapid strong pulse d) Rapid weak pulse e) Cool clammy skin f) Body temperature 102 F {38.9 C)

a) Disorientation b) Red/dry skin c) Rapid strong pulse

An athlete successfully progresses through six months of rehabilitation following a Bankart repair. Which of the following would indicate that the athlete is ready to return to play? Choose all that apply: a) External rotation of the involved shoulder within five degrees of the uninvolved shoulder b) No palpable tenderness in the involved shoulder region c) NSAID use for preventative pain relief before activity d) Pain-free ROM with overhead activities e) Shoulder brace use to limit shoulder abduction and internal rotation f) Strength demonstrated at 80 percent in the involved shoulder as compared to uninvolved shoulder

a) External rotation of the involved shoulder within five degrees of the uninvolved shoulder b) No palpable tenderness in the involved shoulder region d) Pain-free ROM with overhead activities

A hockey player reports to the athletic trainer complaining of right hip pain. The athlete indicates that the pain began approximately three weeks ago and is progressively increasing. The athlete has no prior history of hip injuries. The athletic trainer completes an initial evaluation of this athlete. Initial observations: • No atrophy or ecchymosis present • Right iliac crest and ASIS appear inferior • Pronated feet/ pes planus • Genu Varum • Apparent weight shift to the left side The athlete's leg length has been measured to further complete this evaluation. Assessment of leg length reveals: • 5 mm {0.2 in) difference in true/anatomical leg length measurements • 15 mm {0.9 in) difference in apparent/functional leg length measurements To complete the evaluation, which of the special tests would be most appropriate at this time? Choose all that apply: a) FABER test b) Piriformis test c) Single leg balance d) Supine long sit e) Thomas test f) Thompson test

a) FABER Test b) Piriformis Test e) Thomas Test

Which of the following sports is considered unsafe for an athlete absent one kidney? Choose One: a) Field Hockey b) Gymnastics c) Golf d) Tennis e) Track and Field

a) Field Hockey

When cleaning up dried blood, an athletic trainer is at greatest risk for contracting which of the following? Choose One: a) Hepatitis B b) HIV c) Syphilis d) Tinea Corporis e) Tuberculosis

a) Hepatits B

Which of the following are true regarding emergency action plans? Choose all that apply: a) Must be distributed to athletic trainers and team physicians b) Must be written documents c) Outlines qualifications of those writing the plans d) Should be reviewed and researched monthly e) Should be specific to the activity venue

a) Must be distributed to athletic trainers and team physicians b) Must be written documents e) Should be specific to the activity venue

Upon visual inspection, an athletic trainer observes an athlete's fifth digit in extension of the MCP and DIP joints and flexion of the PIP joint. Which injury has the athlete sustained? Choose One: a) boutonniere deformity b) claw finger c) mallet finger d) swan neck deformity e) trigger finger

a) boutonniere deformity

If the patient is experiencing diplopia, what cranial nerve is affected? Choose One: a) cranial neve II (optic) b) cranial nerve Ill (oculomotor) c) cranial nerve IV (trochlear) d) cranial nerve V (trigeminal) e) cranial nerve VI (abducens)

a) cranial neve II (optic)

62. An athlete complains that his helmet is rocking anteriorly and cutting the bridge of his nose. Which of the following methods would be the best approach to alleviate this problem? Choose One: a) fit the athlete with a smaller helmet b) insert internal padding to the helmet c) insert larger cheek/jaw pads into the helmet d) insert larger ear pads into the helmet e) instruct the athlete to tighten the chin straps

a) fit the athlete with a smaller helmet

In order to prevent the spread of methicillin-resistant staphylococcus aureus (MRSA), how often should whirlpools and turbines should be emptied and thoroughly disinfected? Choose One: a) After every patient b) At the beginning of each day c) At the end of each day d) Once daily e) Once weekly

a. After every patient

A patient reports a positive status for sickle cell trait on the medical history of a PPB. Which of the following actions should the athletic trainer take to prevent a sickle cell event? Choose all that apply: a. Allow longer rest and recover periods between repetitions b. Exclude athlete from one rep max strength testing c. Exclude athlete from participation in timed performance tests (ie: mile run ) d. Monitor athlete closely during competition in high altitude environments e. No modifications to activity are indicated f. Restrict athlete from competing in contact sports

a. Allow longer rest and recover periods between repetitions c. Exclude athlete from participation in timed performance tests (ie: mile run ) d. Monitor athlete closely during competition in high altitude environments

What anatomical position is maintained with the application of a night splint for plantar fasciitis? Choose one: a. Ankle dorsiflexion b. Ankle plantarflexion c. Subtalar eversion d. Subtalar inversion

a. Ankle dorsiflexion

An athlete appears very irritable before an evening basketball game. The coach indicated that the athlete performed very poorly during the pre-game shooting practice. Examination by the athletic trainer reveals that the athlete is agitated, short of breath, lightheaded, nauseated, and sweating profusely. What condition could be indicated by these symptoms? Choose one: a. Anxiety b. Bipolar disorder c. Depression d. Obsessive- compulsive disorder e. Seasonal affective disorder

a. Anxiety

Which of the following is defined as a temporary cessation of breathing? Choose one: a. Apnea b. Bradypnea c. Dyspnea d. Hyperventilation e. Tachypnea

a. Apnea

An athlete presents with a hyphema. Which of the following actions should the athletic trainer take at this time? Choose all that apply: a. Apply an ice compress to the athlete's eye b. Evaluate visual acuity with Snellen eye chart c. Perform Fluorescein dye stain e. Place the athlete in a supine position with elevated legs f. Refer the athlete to physician/emergency room g. Send the athlete home with an eye patch

a. Apply an ice compress to the athlete's eye d. Place the athlete in a recumbent position f. Refer the athlete to physician/emergency room

Which of the following elements should be considered for a pre-event meal? Choose all that apply: a. Avoid foods rich in fiber b. Eat one to two hours prior to the event c. Eat three to four hours prior to the event d. Focus on high- calorie liquid meals e. Focus on meal replacement bars f. Meal should consist of mostly carbohydrates g. Meal should consist of mostly proteins

a. Avoid foods rich in fiber c. Eat three to four hours prior to the event f. Meal should consist of mostly carbohydrates

Which of the following instructions/techniques are correct for cane use? Choose all that apply: a) Cane height is adjusted so elbow is flexed 20-30 degrees b) Cane height is adjusted to the ASIS c) Cane is held in contralateral hand d) Cane is held in ipsilateral hand e) Cane moves forward with the involved extremity f) Cane moves forward with the uninvolved extremity

a. Cane height is adjusted so elbow is flexed 20-30 degrees c. Cane is held in contralateral hand e. Cane moves forward with involved extremity

An athletic trainer has been charged with writing a MRSA management plan regarding MRSA incidents for an area high school athletic department. Which of the following agencies and/or federal regulatory acts would be the most appropriate sources to develop this plan? Choose all that apply. a. Centers for disease control and prevention b. Family educational rights and privacy act c. Health insurance portability and accountability act d. Local health department e. Local American red cross f. Occupational safety and health administration

a. Centers for disease control and prevention d. Local health department f. Occupational safety and health administration

When applying butterfly closures to a laceration, which of the following steps should be performed? Choose all that apply: a. Clean and inspect the wound from inside out b. Clean and inspect the wound from the outside in c. Dry the edges and surrounding skin d. Overlap the skin slightly at edges e. Secure center of wound with butterfly closure and work toward the outside edges f. Secure outside edge of wound working toward the center of wound

a. Clean and inspect the wound from inside out c. Dry the edges and surrounding skin e. Secure center of wound with butterfly closure and work toward the outside edges

To prevent dehydration, athletic trainers should encourage participants to ingest fluids to maintain their urine to which color? Choose one: a. Clear to light yellow b. Brown c. Dark yellow d. Yellow

a. Clear to light yellow

Which of the following nutritional practices would help to decrease the risk of dehydration while exercising? Choose one: a. Decrease caffeine intake b. Decrease potassium intake c. Decrease protein intake d. Increase carbohydrate intake

a. Decrease caffeine intake

To prevent the occurrence of an ulnar collateral ligament sprain in youth baseball players, which of the following strategies is most appropriate? Choose all that apply: a. Develop sound throwing mechanics b. Limit pitch count c. Throw multiple types of pitches d. Throw only curve and breaking pitches e. Train year round

a. Develop sound throwing mechanics b. Limit pitch count

How soon following surgical debridement of the triangular fibrocartilage complex should ROM exercises begin? choose one: a. Immediately as tolerated b. Four weeks post- surgery c. Six weeks post- surgery d. Three weeks post-surgery e. 12 weeks post- surgery

a. Immediately as tolerated

Which of the following signs would indicate a traumatic head injury ? choose all that apply: a. Otorrhea b. Positive halo sign c. Positive transillumination d. Positive weber test e. Rhinorrhea f. Tinnitus

a. Otorrhea b. Positive halo sign e. Rhinorrhea

When applying a splint to an athlete who has been diagnosed with a mallet finger, which of the following would be most appropriate? Choose one: a. Place the DIP joint in full extension b. Place the DIP joint in 30 degrees od flexion c. Place the DIP and PIP joints in 30 degrees of flexion d. Place the PIP joint in 45 degrees of flexion e. Place the PIP joint in full extension

a. Place the DIP joint in full extension

Which of the following signs would result in a Glasgow coma score of 15? Choose all that apply a. Squeezes hand when instructed b. Responds only with painful stimuli c. Opens eyes and looks at the athletic trainer upon approach d. Displaces decerebate posturing e. Displays confusion and disorientation when answering questions f. Answers all questions coherently

a. Squeezes hand when instructed c. Opens eyes and looks at the athletic trainer upon approach f. Answers all questions coherently

19. During an evaluation of an athlete who is two weeks post-ACL surgery, the athletic trainer notes the following information: • AROM: 11-92 degrees • Limited VMO activation • Mild effusion • Pain: rated as 2 on a 10-point scale Which of the following treatments/exercises are most appropriate at this time? Choose all that apply: a) Weighted prone hangs b) Standing terminal knee extensions c) PNF stretches of the hip flexor group d) PNF stretches of the hamstring group e) Patellar mobilizations f) High-volt: monophasic setup, continuous duty cycle

a. Weighted prone hangs d) PNF stretches of the hamstring group e. Patellar mobilizations

During the evaluation of an existing emergency action plan, which components should be reviewed and updated by the athletic trainer? Choose all that apply: a) contact information b) emergency c) equipment d) insurance e) personnel f) qualifications g) transportation h) weather considerations

b )emergency c) equipment e) personnel g) transportation

Ecchymosis around the mastoid secondary to head trauma is indicative of which of the following injuries? Choose One: a) Nasal fracture b) Basilar skull fracture c) C4 vertebral dislocation d) Orbital blowout fracture e) Tempermandibular joint dislocation

b) Basilar skull fracture

Which of the following is the most likely cause of Commotio Cordis? Choose One: a) Dehydration b) Blunt trauma to the chest c) Viral infection in the heart d) Puncture of the heart by a rib e) Fluid accumulation around the heart

b) Blunt trauma to the chest

A runner at a road race collapses at the finish line and is brought to the medical tent for evaluation and treatment. Which of the following signs and symptoms are most indicative of exertional heat stroke? Choose all that apply: a) Body core temperature between 97-102 degrees Fahrenheit (36-38 degrees Celsius) b) Body core temperature greater than 104 degrees Fahrenheit {40 degrees Celsius) c) Cool and clammy skin d) Disoriented e) Hot and wet or dry skin f) Irritable/irrational

b) Body core temperature greater than 104 degrees Fahrenheit {40 degrees Celsius) d) Disoriented e) Hot and wet or dry skin

Total immersion of the body in cold water dissipates heat by which of the following mechanisms? Choose One: a) Changes in hydrostatic pressure b) Conduction of body heat into the cold water c) Decrease in metabolic heat production d) Restoration of blood volume

b) Conduction of body heat into the cold water

Which of the following are the most common symptoms of mild hyperthermia ? Choose One: a) Confusion, dizziness, slurred speech, and agitation b) Confusion, shivering, slurred speech, and apathy c) Confusion, shivering, unsteady gait, and apathy d) Dizziness, shivering, slurred speech, and agitation e) Dizziness, slurred speech, unsteady gait, and agitation

b) Confusion, shivering, slurred speech, and apathy

An athlete presents with the following signs and symptoms : - Recently diagnosed concussion - Severe headache - Nausea - Sleepiness - Unequal pupils What condition should the athletic trainer suspect? Choose One: a) Cerebral contusion b) Epidural hematoma c) Malignant brain edema syndrome d) Stroke

b) Epidural hematoma

How often should the electrical modalities used in an athletic training facility be calibrated in order to reduce the risk of a lawsuit? Choose One: a) When issues arise b) Every year c) Every two years d) Every three years

b) Every year

A football player is diagnosed with myositis ossificans on the anterior aspect of the mid-shaft of the humerus. What would be the most appropriate method to protect the area from further trauma? Choose One: a) Fabricate a donut of open-cell (low density) padding that surrounds the area and is overlaid with a firm plastic material b) Fabricate a donut of closed-cell material (high density) surrounding the area and overlaid with firm plastic material c) Protect the area with 1- to 2-inch (2.5 to 5 cm) thick open-cell (low density) donut padding d) Protect the area with 1to 2 inch (2.5 to 5 cm) closed-cell (high density) padding without a donut e) Protect the area with 1to 2 inch (2.5 to 5 cm) closed-cell (high density) donut Padding

b) Fabricate a donut of closed-cell material (high density) surrounding the area and overlaid with firm plastic material

Which of the following regulates how and when an athlete's private health information can be shared among healthcare professionals? Choose One: a) Occupational Safety and Health Administration b) Health Insurance Portability and Accountability Act c) Family Educational Rights and Privacy Act d) Buckley Amendment

b) Health Insurance Portability and Accountability Act

Which of the following are included in the five cardinal signs of inflammation? Choose all that apply: a) Atrophy b) Heat c) Loss of function d) Paresthesia e) Point tenderness f) Swelling

b) Heat c) Loss of function f) Swelling

While attempting to bunt a pitch, a baseball player is struck in the eye by a ball. The athlete complains of photophobia, loss of visual field, pain, and increased pressure in the eye. Examination by the athletic trainer reveals an accumulation of blood in the eye. Which of the following injuries may be indicated? Choose One: a) Corneal abrasion b) Hyphema c) Orbital fracture d) Retinal detachment e) Traumatic iritis

b) Hyphema

Which statement would be included in the subjective section of the SOAP notes? Choose One: a) First degree (knee) MCL sprain b) I felt a sharp twinge in the back of my thigh c) The athlete has limited active ROM d) The athlete has a positive Lachman's sign on the left knee e) The athlete will have no contact for three days

b) I felt a sharp twinge in the back of my thigh

Which exercise type is designed specifically to match the torque- generating capability of a concentric contracting muscle group throughout the full range of motion? Choose One: a) Isotonic b) lsokinetic c) Isometric d) Plyometric e) Pneumatic

b) Isokinetic

36. What information best defines the roles and responsibilities of an athletic trainer within a specific employment setting? Choose One: a) Employment contract b) Job description c) Performance evaluation d) Policy and procedure manual

b) Job description

An athletic trainer budgets by allocating money between various categories including telephone, office supplies, equipment maintenance, and expendable supplies. What type of budgeting technique is this athletic trainer using? Choose One: a) Fixed b) Line-item c) Performance d) Variable e) Zero-based

b) Line-item

What budget type requires an athletic trainer to list anticipated expenditures for specific categories of program sub- functions, including equipment repair, insurance, and expendable supplies? Choose One: a) Fixed b) Line-item c) Lump-sum d) Performance e) Zero-based

b) Line-item

40. In an acute injury situation, which of the following conditions would necessitate immediate activation of EMS? Choose One: a) Swelling and ecchymosis to a distal extremity articulation b) Loss of vascular supply and neurological integrity distal to an obvious fracture c) Heat cramps that may require rehydration d) Head injury with no associated loss of consciousness e) Bleeding that may require sutures

b) Loss of vascular supply and neurological integrity distal to an obvious fracture

Which of the following are contraindications for the application of spinal traction? Choose all that apply: a) Facet joint hypermobility b) Osteoporosis c) Paraspinal muscle strain d) Spinal ligament contracture e) Spondylolisthesis

b) Osteoporosis e) Spondylolisthesis

What is the recommended time period, in years, that every NOCSAE-approved helmet should be returned to the manufacturer for recertification/ reconditioning? Choose One: a) One year b) Two year c) Three year d) Four year e) Five year

b) Two year

An athlete on the wrestling team presents to the athletic trainer complaining of cramps in his calves. The athlete states that he was running for approximately one hour in order to lose weight. Upon observation, the athletic trainer notes that the athlete seems disoriented and lack balance. As the athlete removes his plastic suit, the athletic trainer notices that the athlete's skin is pale and clammy. In addition, the athlete has an elevated pulse. Based on these signs and symptoms, which of the following conditions is most likely? Choose One: a) common cold b) heat exhaustion c) heat stroke d) hypothermia e) influenza

b) heat exhaustion

65. Following the implementation of an emergency action plan, the athletic trainer documents the event and all related activities.What document is the athletic trainer completing? Choose One: a) assumption of risk form b) incident report form c) medical record d) SOAP note

b) incident report form

Which of the following splinting devices should an athletic trainer use to best immobilize an open fracture? Choose One: a) air splint b) rigid splint c) fiberglass cast material d) plaster cast material e) traction splint

b) rigid splint

Weakness or lack of neuromuscular control of which of the following muscle groups contributes to scapular dyskinesis? Choose all that apply: a) upper trapezius b) serratus anterior c) rhomboids d) middle trapezius e) lower trapezius

b) serratus anterior

What angle in degrees of knee flexion will most likely elicit pain during a positive Noble compression test? Choose one: a. 0 b. 30 c. 60 d. 90 e. 120

b. 30

Which of the following best describe a 1 MHz thermal ultrasound application? Choose all that apply: a. Beam is relatively collimating b. Beam is relatively divergent c. Penetrates up to 5 cm deep d. Penetrates up to 1.5 cm deep e. Retains heat half as long as 3 MHz f. Retains heat twice as long as 3 MHz

b. Beam is relatively divergent c. Penetrates up to 5 cm deep f. Retains heat twice as long as 3 MHz

During a soccer game, the athletic trainer sees a flash of lighting above the playing field and it begins to rain. What is the appropriate course of action? Choose one: a. Clear the field and postpone the game until the flash bang ration exceeds 20 seconds b. Clear the field and postpone the game for at least 30 minutes c. Allow the game to proceed, but only if the flash band ratio exceeds 30 seconds d. Allow the game to proceed, but encourage fans to stay out of the bleachers

b. Clear the field and postpone the game for at least 30 minutes

Which term best describes the cumulation of personal values, preferences, experiences and wisdom when using clinical judgement? Choose one: a. Clinical-based outcomes b. Clinical expertise c. Clinical research d. Double blind studies e. Patient preference

b. Clinical expertise

What is the main function of a rigid foot orthotic? Choose one: a. Absorb shock b. Control rearfoot motion c. Decrease ankle dorsiflexion d. Prevent metatarsal fractures

b. Control rearfoot motion

A 19 year old basketball player was poked in the eye with a finger during a rebound. The player complains of pain, diplopia, floaters, and photophobia. Upon inspection, the athletic trainer notes the following: • Subconjunctival redness • Pupils equal and reactive • Excessive tearing • Eye tracking normal The athlete's condition was evaluated using a fluorescein strip. Which of the following conditions would be ruled out using this application? Choose one: a. Acute conjunctivitis b. Corneal abrasion c. Hyphema d. Retinal detachment e. Ruptured globe

b. Corneal abrasion

A 19 year old basketball player was poked in the eye with a finger during a rebound. The player complains of pain, diplopia, floaters and photophobia. Upon inspection, the athletic trainer notes the following: • Subconjunctival redness • Pupils equal and reactive • Excessive tearing • Eye tracking normal Based on the findings found in the initial evaluation, the athletic trainer would consider which of the following conditions in their differential diagnosis? Choose all that apply: a. Acute conjunctivitis b. Corneal abrasion c. Hyphema d. Retinal detachment e. Ruptured globe

b. Corneal abrasion d. Retinal detachment

Which of the following are true regarding the use of cryotherapy? Choose all that apply: a. Decreases ischemia b. Decreases muscle spasm c. Decreases nerve conduction velocity d. Increases depolarization threshold e. Increases oxygen consumption f. Increases the rate of cell metabolism

b. Decreases muscle spasm c. Decreases nerve conduction velocity d. Increases depolarization threshold

Which of the following is a non-thermal effect of shortwave diathermy? Choose One: a. Decreased joint stiffness b. Enhance tissue healing c. Hematoma absorption d. Increased collagen extensibility e. Vasodilation

b. Enhance tissue healing

Which of the following design elements in an athletic training facility is most likely to be influenced by the americans with disabilities act (ADA)? Choose one: a. Ceiling height b. Entryway c. Lighting d. Ventilation system e. Ground fault interrupter circuit

b. Entryway

After sustaining a displaced femoral fracture, injury to which anatomical structure could result in a possible catastrophic injury? Choose One: a. Adductor magnuis b. Femoral artery c. Femoral nerve d. Inguinal ligament e. Pubic symphysis

b. Femoral artery

A hockey player reports to the athletic trainer complaining of right hip pain. The athlete indicates that the pain began approximately three weeks ago and is progressively increasing. The athlete has no prior history of hip injuries. The athletic trainer completes an initial evaluation of this athlete. Initial observations: • No atrophy or ecchymosis present • Right iliac crest and ASIS appear inferior • Pronated feet/ pes planus • Genu Varum The athlete's leg length has been measured to further complete this evaluation. Assessment of leg length reveals: • 5 mm (0.2 in) difference in true/anatomical leg length measurements • 15 mm (0.9 in) difference in apparent/functional leg length measurements Which of the following findings are correct interpretations of the measurements? Choose all that apply. a. Functional leg length difference is negative b. Functional leg length difference is positive c. The athlete has tightness and/or weakness of muscles or joint structures. d. There is a possible disruption to the growth plate.

b. Functional leg length difference is positive c. The athlete has tightness and/or weakness of muscles or joint structures.

An athlete's parent has asked what are the most common factors/causes associated with her child's temporomandibular joint dysfunction. Which of the following would be included in the explanation to the parent? Choose all that apply: a. Chronic sinus congestion b. Grinding teeth c. Poor approximation of the teeth d. Tonsillitis e. Vertigo

b. Grinding teeth c. Poor approximation of the teeth

Which of the following are signs and symptoms for meningitis? Choose all that apply: a. Diplopia b. High fever c. Negative Kernig's sign d. Severe headache e. Slow onset of symptoms f. Stiff neck

b. High fever d. Severe headache f. Stiff neck

A 19 year old basketball player was poked in the eye with a finger during a rebound. The player complains of pain, diplopia, floaters, and photophobia. Upon inspection, the athletic trainer notes the following: • Subconjunctival redness • Pupils equal and reactive • Excessive tearing • Eye tracking normal Which of the following medical specialists would be most qualified to treat this athlete and make a return to play decision? Choose one: a. Neurologist b. Ophthalmologist c. Optician d. Optometrist e. Otolaryngologist

b. Ophthalmologist

Which term is used for a fracture in which the distal radius and ulna are displaced in a volar direction? Choose one: a. Thomas b. Smith c. Jones d. Boxer's e. Bennett's

b. Smith

Which of the following routes of delivery is used when administering topical medication? Choose one: a. Sublingual b. Subcutaneous c. Oral d. Intramuscular e. Inhalation

b. Subcutaneous

How soon following exercise should carbohydrates be consumed to replenish glycogen stores? Choose one: a. Within the first 30 minutes b. Within the first four hours c. Within the first 12 hours d. Within the first 24 hours e. Within the first 48 hours

b. Within the first four hours

What is the time frame in days for a conditioned athlete to achieve acclimatization in the prevention of heat illness? Choose One: a) 1-4 b) 5-9 c) 10-14 d) 15-19 e) 20-25

c) 10-14

A hockey player reports to the athletic trainer complaining of right hip pain. The athlete indicates that the pain began approximately three weeks ago and is progressively increasing. The athlete has no prior history of hip injuries. The athletic trainer completes an initial evaluation of this athlete. Initial observations: • No atrophy or ecchymosis present • Right iliac crest and ASIS appear inferior • Pronated feet/ pes planus • Genu Varum • Apparent weight shift to the left side Manual muscle testing of this athlete's hip revealed weakness in the following muscles: obturator externus, gemellus superior, piriformis. Which movement would be limited for this athlete? Choose One: a) Abduction b) Adduction c) External rotation d) Flexion e) Internal rotation

c) External Rotation

Which of the following is a primary benefit of goal setting with an athlete undergoing rehabilitation? Choose One: a) Provide alternatives for physician's protocol b) Improve communication with the coach c) Focus the athlete's attention and actions d) Establish a date for the athlete to return to play e) Decrease tissue healing time

c) Focus the athlete's attention and actions

Which of the following can be palpated on the ventral/palmar surface of the palm, proximal to the fifth digit? Choose One: a) Anterior compartment b) Central compartment c) Hypothenar eminence d) Lateral palmar compartment e) Thenar eminence

c) Hypothenar eminence

Which of the following is the most restrictive form of credential regulation? Choose One: a) Certification b) Exemption c) Licensure d) Registration

c) Licensure

An athlete presents with a suspected retinal detachment. To whom should this athlete be referred? Choose One: a) Otolaryngologist b) Orthoptist c) Ophthalmologist d) Neurologist

c) Ophthalmologist

Anteriorly rounded shoulders in a swimmer might indicate tightness in which of the following muscles? Choose One: a) Deltoid b) Latissimus dorsi c) Pectoralis minor d) Rhomboid major e) Trapezius

c) Pectoralis minor

During a foraminal compression test (Spurling's test) of the cervical spine, the examiner carefully presses on which location? Choose One: a) Base of the skull b) Occipital protuberance c) Top of the head d) Zygomatic arch

c) Top of the head

The athletic trainer is summoned to the soccer field for an unconscious athlete. Which of the following are part of the primary exam? Choose all that apply: a. Assess the pupil response b. Assess the dermatomes c. Check for a pulse d. Check for profuse bleeding e. Check the airway f. Look for gross deformity

c. Check for a pulse d. Check for profuse bleeding f. Look for gross deformity

Which two ligaments combine to form the coracoclavicular ligament? Choose one: a. Conoid and trapezium b. Conoid and trapezius c. Conoid and trapezoid d. Coracoid and humeral e. Coracoid and trapezoid

c. Conoid and trapezoid

During a pre-participation examination, an athlete reveals that they had surgery during the summer that may affect their sports participation status. The athletic trainer contacts the surgeon to learn the details of the surgery, but is denied access to the athlete's records. What federal legislation is responsible for the denied request? Choose one: a. Americans with disabilities act b. Family educational rights and privacy act c. Health insurance portability and accountability act d. Title IX, education amendments of 1972

c. Health insurance portability and accountability act

Acute extertional rhabdomyolysis is most likely to occur in which environmental conditions? Choose one: a. Cold and dry b. High altitude c. Hot and humid d. Polluted areas

c. Hot and humid

During a pre-game meal, an athlete displays difficulty breathing and falls to the floor. The athletic trainer's primary survey reveals the athlete is unconscious with a weak pulse, tachycardia, and apnea. What is the most appropriate immediate action for the athletic trainer to take? Choose one: a. Apply AED b. Initiate CPR c. Initiate airway management d. Obtain vital signs e. Treat for shock

c. Initiate airway management

Which of the following methods of non- verbal communication can help reinforce positive verbal communication? Choose all that apply: a. Finger tapping b. Keeping arms crossed c. Making eye contact d. Neat, well-fitted clothing e. Polite hand shake f. Turning your body away from the conversation

c. Making eye contact d. Neat, well-fitted clothing e. Polite hand shake

A patient who is two weeks post-surgery for hip replacement complains of muscle stiffness. Which of the following modalities would be best to use in order to heat the tissues prior to stretching exercises? Choose One: a) Friction massage b) Laser c) Moist hot pack d) Pulsed ultrasound e) Warm whirlpool

c. Moist hot pack

A 19 year old basketball player was poked in the eye with a finger during a rebound. The player complains of pain, diplopia, floaters, and photophobia. Upon inspection, the athletic trainer notes the following: • Subconjunctival redness • Pupils equal and reactive • Excessive tearing • Eye tracking normal Five minutes later, the athlete complains of loss of peripheral vision. Based on these findings, what action should be performed next? Choose one: a. Evaluate visual acuity with a Snellen eye chart b. Irrigate the eye with saline solution c. Patch both eyes and transport to the emergency room d. Patch injured eye and place in recumbent position e. Perform cranial nerve assessment for cranial nerves II(optic), IV(trochlear), and VI (abducens)

c. Patch both eyes and transport to the emergency room

Which of the following documents is essential for the treatment of participants who are minors? Choose one: a. Current medical progress notes from a licensed physician b. Current physical examination results from a licensed physician c. Permission/consent for medical treatment from a parent or guardian d. Written assumption of risk signed by parent of guardian

c. Permission/consent for medical treatment from a parent or guardian

What are the appropriate procedures to follow when cleaning bloodborne pathogens from wrestling mats? Choose all that apply: a. Clean area with soap and water b. Disinfected contaminated area with alcohol based sanitizer c. Place soiled materials in waste receptacle d. Place saturated materials in biohazard trash e. Spray area with alcohol and water solution f. Spray contaminated area with 1:10 bleach solution

c. Place soiled materials in waste receptacle d. Place saturated materials in biohazard trash f. Spray contaminated area with 1:10 bleach solution

Which of the following budget types requires an explanation for each item purchased? Choose one: a. Fixed b. Spending-ceiling c. Zero-based d. Line-item e. Lump-sum

c. Zero-based

75. What is the optimal position for cervical spine traction to increase the intervertebral disc space? Choose One: a) 0 degrees to 10 degrees extension b) 0 degrees to 10 degrees flexion c) 20 degrees to 30 degrees extension d) 20 degrees to 30 degrees flexion e) Neutral position

d) 20 degrees to 30 degrees flexion

An outdoor lacrosse practice has been suspended due to a thunderstorm. How many minutes should pass after the last sound of thunder or lightning strike before practice is resumed? Choose One: a) 15 b) 20 c) 25 d) 30 e) 35

d) 30

When splinting a suspected fracture, which of the following is most important for proper splint application? Choose One: a) Reduce the fracture prior to splinting b) Immobilize only the joint proximal to the fracture c) Check proximal neurovascular function d) Check distal neurovascular function

d) Check distal neurovascular function

While evaluating an athlete's hip, an athletic trainer notes an 18-degree angle of torsion, femoral rotation, squinting patellae, and toe-in gait. Visually, this indicates which of the following conditions? Choose One: a) Coxa valga b) Coxa vara c) Femoral torsion d) Hip anteverslon e) Hip retroversion

d) Hip anteverslon

After assessing an athlete with a suspected femoral fracture, the athlete appears pale, is perspiring, has a rapid and weak pulse, and has rapid and shallow respirations. Which physiological response is most likely occurring? Choose One: a) Blood pooling due to vasoconstriction b) Decreased heart rate due to hemorrhaging c) Hypertension due to hypervolemia d) Hypovolemia due to hemorrhaging e) Increased cardiac output due to decreased peripheral resistance

d) Hypovolemia due to hemorrhaging

Which of the following infections usually develops vesicles? Choose One: a) Scabies b) Ringworm c) Plantar warts d) Impetigo e) Athlete's foot

d) Impetigo

A fixed amount of money without designation defines which type of budgeting? Choose One: a) Zero-based b) Variable c) Performance d) Lump-sum e) Line-item

d) Lump-sum

In the event of a death, which of the following terms refers to the act of the institution failing to have emergency planning in place? Choose One: a) Assumption of risk b) Malfeasance c) Misfeasance d) Nonfeasance e) Sovereign immunity

d) Nonfeasance

Which of the following medical conditions discovered during a pre- participation examination would disqualify an athlete from participation? Choose One: a) Carditis b) Diabetes c) Functional instability of the ankle d) Previous episode of a heat stroke e) Spondylolysis

d) Previous episode of a heat stroke

A rower reports to the athletic trainer complaining of pain with repetitive wrist motions. Examination reveals that the athlete has swelling over the extensor carpi radialis longus and has difficulty with passive stretching. What action should the athletic trainer take to manage this athlete's condition? Choose One: a) recommend the athlete go to the emergency room for X-rays b) recommend the athlete decrease activity and use moist heat two times a day c) recommend the athlete increase activity and stretch regularly prior to exercise d) recommend the athlete decrease activity and use ice massage three times a day

d) recommend the athlete decrease activity and use ice massage three times a day

Which of the following is an appropriate temperature for an ice immersion treatment? Choose one: a. 25 degrees F (-3.8 degrees C) b. 35 degrees F (1.6 degrees C) c. 45 degrees F (7.2 degrees C) d. 55 degrees F (18.3 degrees C) e. 65 degrees F (18.3 degrees C)

d. 55 degrees F (18.3 degrees C)

A 16 year old athlete with Type I diabetes complains of lightheadedness and muscle weakness, and is found to have a blood glucose reading of 63 ml/dl. Which of the following actions should the athletic trainer take to manage the athlete? Choose all that apply: a. Activate EMS b. Administer epinephrine injection c. Administer 4 oz (113 g) of diet cola d. Administer 4 oz (113.4 g) of orange juice e. Administer glucagon injection f. Contact parents of guardian g. Re-check blood glucose in 15 minutes h. Remove from activity

d. Administer 4 oz (113.4 g) of orange juice f. Contact parents of guardian g. Re-check blood glucose in 15 minutes h. Remove from activity

What is the optimal arm position when taking a patient's blood pressure? Choose one: a. Arm at side 12th rib with elbow bent b. Arm at side with elbow straight c. Arm held horizontal above 12th rib with elbow bent d. Arm relaxed with elbow and forearm level with heart e. Horizontal at shoulder level with elbow straight

d. Arm relaxed with elbow and forearm level with heart

How often should a patient take a prescription medicine prescribed as QID? Choose one: a. As needed b. Every day c. Every other day d. Four times a day e. Three times a day

d. Four times a day

While evaluating an athlete's hip, an athletic trainer notes an 18 degree angle of torsion, femoral rotation, squinting patellae, and toe-in gait. Visually, this indicates which of the following conditions? Choose one: a. Coxa valga b. Coxa vara c. Femoral torsion d. Hip anteversion e. Hip retroversion

d. Hip anteversion

Which of the following PNF stretches involves an isometric contraction of the hamstring muscle group during the push phase followed by a passive stretching of the hamstring muscle group in the relaxation phase? Choose one: a. Agonist contraction b. Antagonist contraction c. Contract-relax d. Hold- relax e. Slow-reversal-hold-relax

d. Hold- relax

Which of the following adjustments to a biphasic waveform would cause an increase to the electrical current? Choose all that apply: a. Change the polarity b. Decrease the amplitude c. Decrease the frequency d. Increase the amplitude e. Increase the hz f. Increase the pulse width g. Increase the ramp size

d. Increase the amplitude e. Increase the hz f. Increase the pulse width

Which of the following patient rated outcome measures would be most appropriate to utilize for a patient with patella femoral pain syndrome? Choose all that apply: a. Bilateral active and passive knee range of motion b. Bilateral hip and knee MMT c. Foot and ankle ability measure (FAAM) d. International Knee Documentation Committee Knee Form (IKDC) e. Lower Extremity Functional Scale (LEFS)

d. International Knee Documentation Committee Knee Form (IKDC) e. Lower Extremity Functional Scale (LEFS)


Related study sets

Organizational Behavior Final Study Guide

View Set

ATI RN Nursing Care of Children Practice B

View Set

geography of N. america quiz questions

View Set

Chapter 11: The Muscular System 11.1-11.3

View Set

MD33000 필수어휘 3/6 (DE~EX) 예문, 파생어

View Set